Download list of contents - Queen`s University Belfast

Document related concepts

Health equity wikipedia , lookup

Adherence (medicine) wikipedia , lookup

Patient safety wikipedia , lookup

Medical ethics wikipedia , lookup

Electronic prescribing wikipedia , lookup

Transcript
PRIMARY HEALTHCARE
STUDY GUIDE
Department of General Practice
2006 – 2007
Staff Members
Head of Department
Professor Philip Reilly
Senior Lecturers
Dr Keith Steele
Dr Drew Gilliland
Dr Kieran McGlade
Dr Margaret Cupples
GP Research Fellows
Dr Nigel Hart
Dr Katherine MacLurg
Research Registrars
Dr Naoimh White
Dr Waqar Ahmed
Departmental Secretary
Miss Cathleen Agnew
Additional Staff
Mrs Claire Leathem
Dr Mark Tully
Mrs Louise Seaye
LIST OF CONTENTS
Primary Health Care Teaching Programme 2006/2007
PAGE
1-3
Aims & Objectives
4-5
Learning Outcomes
5-6
Practice Attachments
7-8
Clerkship and Teaching
8 – 12
End Course Assessment
12 – 15
Record Card, Evaluation, Feedback & Attendance
15 - 16
Students in Difficulty
16
Intimate Examination of patients by students
16
Student Support & Guidance Information Sheet
17
Record Card
18 – 22
Practice Experience Questionnaire
23 – 26
Student Mini Curriculum Vitae
27
Self Assessment Questionnaire Information Sheet
28
What is General Practice
29 – 37
The New General Practice Contract
37 - 38
Consultation Skills
39
Consent
40 - 42
Communication in Medicine
43 - 45
Evidence-Based Communication
46 - 50
Audit
51 - 55
Prescribing in General Practice
56 - 67
Common Symptoms & “Red Flags”
68
Medical Records
69 - 71
Management of Common Conditions
72 - 74
Complaints Handling in General Practice
75 - 77
Emergencies
78
Ethics Course
79 - 84
Evidence Based Practice
85
Health Promotion
86 - 94
Child Health in General Practice
95 - 102
Multicultural Issues in Primary Care
103
Modified Essay Questions
104 - 129
DEPARTMENT OF GENERAL PRACTICE, PRIMARY HEALTH
DAY
TIME
Mon 28th Aug 06 =
Bank Hol
25th Dec 06 - 5thJan 07
inc = C’mas Hols
Mon. 7th & 28th May 07
= Bank Hols
a.m.
p.m.
TUESDAY
a.m.
p.m.
WEDNESDAY
THURSDAY
a.m.
a.m.
p.m.
FRIDAY
a.m.
2006-2007
Mon. 4th Sept - Fri. 8th Sept 2006
Mon. 30th Oct - Fri. 3rd Nov 2006
Mon. 11th Sept -Fri. 15th Sept 2006
Mon. 6th Nov -Fri. 10th Nov 2006
Mon. 18th Sept - Fri. 22nd Sept 2006
Mon. 13th Nov - Fri. 17th Nov 2006
Mon. 18th Dec – Fri. 22nd Dec 2006
Mon. 8th Jan - Fri. 12th Jan 2007
Mon. 15th Jan - Fri. 19th Jan 2007
Mon. 22nd Jan - Fri. 26th Jan 2007
Mon. 26th Feb – Fri. 2nd Mar 2007
Tues. 8th May - Fri. 11th May 2007
Mon. 5th Mar - Fri. 9th Mar 2007
Mon. 14th May - Fri. 18th May 2007
Mon. 12th Mar - Fri. 16th Mar 2007
Mon. 21st May -Fri. 25th May 2007
Mon. 19th Mar - Fri. 23rd Mar 2007
Tues. 29th May - Fri. 1st June 2007
1
MONDAY
CARE, TEACHING PROGRAMME
Tues. 29th Aug - Fri. 1st Sept 2006
Mon. 23rd Oct - Fri. 27th Oct 2006
2
AGEING & HEALTH
INTRODUCTION
Venue: D.G.M.
9.15am
PRIMARY HEALTH CARE
INTRODUCTION
DG
CONSULTATION SKILLS KS
Venue: D.G.P.
2.00pm
PRACTICE ATTACHMENTS

3
4




EVIDENCE BASED PRACTICE I
MEC, KS, WA
Venue: D.G.P.
9.15am



AGEING & HEALTH
Venue: D.G.M














2.00pm.
AGEING & HEALTH
Venue: D.G.M
9.15am.
(GROUP 2)
9.15am-10.45am CHILD HEALTH IN
GENERAL PRACTICE
KMcG
11.15-12.45pm COMPUTER
ASSISTED LEARNING
KMcG
Venue: D.G.P.
SIMULATED PATIENTS
DG/KMacL
Venue: D.G.P.
(GROUP 2)
9.15 - 10.45am
EMERGENCIES JW /AN
2.00pm
11.15 - 12.30pm
MANAGEMENT OF ACUTE &
CHRONIC COMMON CONDITIONS
(GROUP 2)
9.15-10.15am ATTACHMENT FEEDBACK
DG & MEC
10.45 - 12.45pm AUDIT
Venue: D.G.P.
JMcC
NW
p.m.
Venue: D.G.P.
CLOSED CIRCUIT TELEVISION
CCTV I PATIENT INTERVIEWS I
2.00pm INNER RING PRACTICES
CLOSED CIRCUIT TELEVISION
CCTV II PATIENT INTERVIEWS II
2.00pm INNER RING PRACTICES

Free
Page 1
DEPARTMENT OF GENERAL PRACTICE, PRIMARY HEALTH CARE, TEACHING PROGRAMME
DAY
TIME
2nd Apr – 13th Apr 07 inc
= Easter Hols
Mon. 2nd Oct - Fri. 6th Oct 2006
Mon. 27th Nov - Fri. 1st Dec 2006
Mon. 5th Feb - Fri. 9th Feb 2007
Mon. 16th Apr – Fri. 20th Apr 2007
Mon. 9th Oct -Fri. 13th Oct 2006
Mon. 23rd Apr -Fri. 27th Apr 2007
Mon. 16th Oct - Fri. 20th Oct 2006
Mon. 11th Dec - Fri. 15th Dec 2006
Mon. 19th Feb - Fri. 23rd Feb 2007
Mon. 30th Apr - Fri. 4th May 2007
Mon. 4th June - Fri. 8th June 2007
Mon. 11th June - Fri. 15th June 2007
Mon. 18th June -Fri. 22nd June 2007
Mon. 25th June - Fri. 29th June 2007
5
MONDAY
a.m.
a.m.
TUESDAY
p.m.
WEDNESDAY
THURSDAY
a.m.
a.m.
Mon. 12th Feb -Fri. 16th Feb 2007
7
8





EVIDENCE BASED PRACTICE II
MEC, KS, WA
Venue: D.G.P
9.15am.



HEALTH PROMOTION
Venue: D.G.P.






Venue: D.G.P.
2.00pm
HANDLING COMPLAINTS
MULTICULTURAL ISSUES
Venue: D.G.P.
DG
MC
PRESCRIBING
Venue: D.G.P.
MT
2.00pm
EQ
9.15am
(GROUP 1)
9.15am-10.45am CHILD HEALTH IN
GENERAL PRACTICE
KMcG
11.15-12.45pm COMPUTER
ASSISTED LEARNING
KMcG
Venue: D.G.P.
SIMULATED PATIENTS
p.m.
Mon. 4th Dec - Fri. 8th Dec 2006
6
PRACTICE
ATTACHMENTS
GENERAL PRACTICE ETHICS
KS/DG
Venue: D.G.P.
9.15am
COMMON SYMPTOMS & “RED FLAGS”
p.m.
2006-2007
Mon. 25th Sept - Fri. 29th Sept 2006
Mon. 20th Nov - Fri. 24th Nov 2006
Mon. 29th Jan - Fri. 2nd Feb 2007
Mon. 26th Mar – Fri. 30th Mar 2007
Venue: D.G.P.
DG/KMacL
2.00pm





FRIDAY
a.m.
(GROUP 1)
9.15 - 10.45am
EMERGENCIES JW /AN
(GROUP 1)
9.15-10.15am ATTACHMENT FEEDBACK
DG & MEC
11.15 - 12.30pm
MANAGEMENT OF ACUTE &
CHRONIC COMMON CONDITIONS
10.45 - 12.45pm AUDIT
JMcC

Venue: D.G.P.
11.15 - 1.00pm GROUP B OSCE
GROUP A
Evaluation/MEQ
NW
Venue: D.G.P.
CLOSED CIRCUIT TELEVISION
CCTV I PATIENT INTERVIEWS I
INNER RING PRACTICES
2.00pm
p.m.
CLOSED CIRCUIT TELEVISION
CCTV II PATIENT INTERVIEWS II
INNER RING PRACTICES
2.00pm
END COURSE ASSESSMENT.
OSCE, ORAL, EVALUATION, MEQ
9.00 - 11.00am GROUP A OSCE
GROUP B
Evaluation/MEQ

3.00pm RESULTS AND FEEDBACK
Venue: DGP
Page 2
PLEASE NOTE:
MONDAY, 28th August 2006: BANK HOLIDAY:- Group D PRIMARY HEALTH CARE INTRODUCTION MOVED TO
9.00AM, TUESDAY, 29th August 2006. Dept of General Practice, Dunluce H.C.
MONDAY, 7th May 2007: BANK HOLIDAY: - Group C PRIMARY HEALTH CARE INTRODUCTION MOVED TO
9.00AM, TUESDAY, 8th May 2006. Dept of General Practice, Dunluce
KEY
PMR
DG
KS
MEC
KMcG
KMacL
WA
NW
JMcC
=
=
=
=
=
=
=
=
=
Professor Philip Reilly
Dr Drew Gilliland
Dr Keith Steele
Dr Margaret Cupples
Dr Kieran McGlade
Dr Katherine MacLurg
Dr Waqar Ahmed
Dr Naiomh White
Dr Jean McClune
JW
AN
MT
DW
EQ
MC
=
=
=
=
=
=
Dr John White
Dr Andy Nelson
Dr Mark Tully
Dr Diane Wilson
Dr Emma Quinn
Dr Mairead Corrigan
D.G.M
=
D.G.P
=
Dept. of Geriatric Medicine,
Whitla Medical Building
Dept. of General Practice,
Dunluce Health Centre
Page 3
4
STUDY GUIDE
FOURTH YEAR COURSE IN PRIMARY HEALTH CARE 2006-2007
General Statement of Aims
The overall aim of the Fourth Year course is to help you to become a better doctor
irrespective of your current career preference or eventual career choice and
specifically to assist students to recognise, adopt and develop those clinical skills
and values that are fundamental to the practice of good caring medicine whatever
the clinical setting. The purpose of this Study Guide is to help you to gain
maximum benefit from your time with us in General Practice.
Before commencing this attachment you will have completed at least three years of
your course and will already have acquired a considerable knowledge base, and
skills and attitudes.
There are a number of areas in which we would expect you to be competent prior
to this attachment.
PRE-REQUISITE OUTCOMES
1.
To obtain a relevant history from an individual using appropriate questions
and responses covering the following systems - cardiovascular, respiratory,
gastro-intestinal, genito-urinary medicine, central nervous system,
peripheral nervous system, musculoskeletal system and skin. This would
include the use of a stethoscope and sphygmomanometer.
2.
To assess a patient by examining competently the following systems cardiovascular, respiratory, genito-urinary, central and peripheral nervous
system, musculoskeletal and skin.
3.
To be able to take a midstream specimen of urine and be able to test for
and recognise abnormalities using a dip stick.
4.
To recognise the normal ear, nose and throat and distinguish a range of
common conditions by history taking and examination including the use of an
auriscope.
5.
To recognise the normal features of the eye on examination and distinguish a
range of common conditions by history taking and examination including the
competent use of an ophthalmoscope.
6.
To define a list of common gynaecological terminology and to identify by history
taking and relevant examination common gynaecological conditions and to be
able to suggest specific management options.
5
7.
To identify children with a range of common paediatric conditions. To state the
various developmental milestones.
8.
To understand the pharmacology and therapeutic uses of commonly used
drugs and be aware of the particular precautions with regard to prescribing in
young children, the elderly and those with liver and renal disease.
LEARNING OUTCOMES
On completion of this course the successful student should be able to:
1.
Identify a patient’s reasons for consulting by relating effectively to the
patient.
2.
Demonstrate good communication skills.
3.
Perform an appropriate physical examination.
4.
Define the likely underlying causes of acute and chronic conditions
commonly encountered in General Practice.
5.
Formulate management plans appropriate to the findings for acute and
chronic conditions commonly encountered in General Practice.
6.
Determine the impact of a patient’s problems/complaints on lifestyle and
interpersonal relationships.
7.
Describe the Primary Healthcare team and the role of each individual
member.
8.
Describe the role of the computer in General Practice.
9.
Interpret medical audit and its application.
10.
Identify opportunities for preventative clinical medicine and health
education in the primary care setting.
11.
Distinguish between good and bad medical records.
12.
Critically appraise a scientific paper.
13.
Develop skills in providing feedback on colleagues’ performance in a
clinical setting using Pendleton’s rules.
14.
Apply an ethical framework to solve ethical dilemmas identified within the
consultation.
6
Common Conditions/Presentations
Students are expected to be familiar with the following common
conditions/presentations in general practice. These will help direct your learning
and may be included in your end of course assessment.
1. Emergency Contraception
2. Combined Contraceptive Pill
3. Sore Ear
4. Back Pain
5. Sore Throat
6. Headache
7. Fever in Child / recognise sick child
8. “Flu” like illness
9. Proven UTI in a child
10. Vomiting & Diarrhoea
11. Impetigo
12. Acne
13. Scabies
14. Head Lice
15. Shingles
16. Nappy Rash
17. Psoriasis
18. Eczema
19. Gout
20. Dizziness
21. Tired All The Time
22. Abdominal Pain
23. Dyspepsia
24. Rectal bleeding
25. Haematuria
26. Asthma /COPD
27. Cough
28. Shortness of breath
29. Cancer
30. Diabetes
31. Hypertension
32. Coronary Heart Disease
33. Chest Pain
34. Heart failure
35. Atrial fibrillation
36. Chronic Kidney Disease
37. Epilepsy
38. Stroke
39. Depression / anxiety
40. Palliative care
41. Obesity
42. Thyroid disease
43. Osteoarthritis
7
PRACTICE ATTACHMENTS
Students are asked to complete a questionnaire about where they want to do their
attachment. While we do try to accommodate students requesting specific practices,
this is not always possible as there are over 150 individual practice attachments. For
reasons of patient confidentiality you cannot be allocated to the practice your family is
registered with. Please let Miss Cathleen Agnew, Departmental Secretary, know
immediately if this occurs.
Usually practices more than 15 miles outside Belfast are residential. You will either
stay in the local hospital, in B & B accommodation or occasionally with the general
practitioner. If you have a car you can opt to travel daily at your own expense. Please
let the practice know if you are not going to need the accommodation.
A list of students and their allocated tutors will be posted on the Departmental notice
board in August. Please contact your GP tutor at least two weeks prior to your
attachment. There is a grant of up to £31.28 per day when using B&B
accommodation. Please pay for the B&B accommodation and give the receipt to Miss
Cathleen Agnew, Departmental Secretary, for reimbursement. You may be able to
ask the proprietor to invoice the Department of General Practice directly and in this
case you give the invoice to Miss Cathleen Agnew who will arrange payment directly
to the proprietor. This takes approximately two weeks. In addition there is a daily
subsistence allowance of £9.96 per day when using B&B accommodation.
There is no re-imbursement for those students staying with relatives, friends or in
hospital accommodation as they will have access to subsidised food.
If the rate is more than £31.28 please get authorisation from Ms Cathleen
Agnew, Department of General Practice, 028 9020 4252.
In very exceptional circumstances, we may be able to change your allocation. You
would need to speak to Dr A Gilliland, Course Co-ordinator, about this.
COURSE READING
Essential
A textbook of General Practice. Edited by Anne Stephenson. 2nd Edition.
Published by Arnold, 2004. Available from Miss C Agnew, Departmental
Secretary. £6.00 deposit (£5.00 refunded after returning the book).
Recommended
Fraser RL. Clinical Methods - A General Practice Approach. 3rd Edition.
Butterworth 1999.
Palmer Notes for the MRCGP. 3rd Edition. Blackwell Scientific Publications.
McWhinney IR. A Textbook of Family Medicine, Oxford University Press.
8
Corney. Developing Communication and Counselling Skills in Medicine.
Routledge.
Reference
British National Formulary
Khot and Polmear. Practical General Practice Guidelines for Logical
Management.
2nd Edition.
Fry. General Practice - The Facts. Radcliffe Medical Press.
Rubeinstein Wayne. Lecture Notes on Clinical Medicine. 6th Edition.
Blackwell Scientific Publications.
THE FOURTH YEAR CLERKSHIP
The fourth year Primary Health Care clerkship is twinned with Ageing and Health. The
block course lasts eight weeks and includes four weeks in each specialty. There are
also two Ethics Seminars in Week 5 of the Block.
While some of the teaching on this course is factual, most is concerned with skills and
attitudes which will enable students to develop coping skills for situations they will be
faced with now and in the future. Activity of this sort is experiential and attendance is
essential if skills are to be acquired.
In your general practice block one week is spent in the department in preparation for the
three-week clerkship with a GP in his/her practice. You will come back to the
Department of General Practice for the Friday of your first week with your GP.
VIDEO SURGERY TEACHING SESSIONS (Two)
Students are allocated in groups of 2-3 to General Practitioners in the Belfast area. A
surgery of 6 patients is booked by the GP. The patients are not pre-selected and
therefore can have a wide range of presenting complaints/symptoms.
The student generally has 15 minutes to take a relevant history and examination where
indicated.
The student is also expected to contribute to how the patient should be managed, eg
simple advice, relevant health promotion, advice on how to take medication and how it
works and the writing of prescriptions.
Each consultation is recorded and afterwards is replayed for analysis by tutor and
student. The tape is erased afterwards. This is not an assessment and it allows students
to see for themselves how others see them.
9
COMPUTER BASED LEARNING RESOURCES
A variety of computer based learning materials are available on a semi-open
access basis in the Department’s computer laboratory. See the Department’s
secretary for assistance if needed in using the packages listed below. The
materials are also available in the Open Access Centre at Mulhouse in the Royal
Victoria Hospital.
- Introduction to prescribing
- Palliative Care in General Practice
- Dermatology in General Practice
- The computerised MEQ
- See separate sheets for a paragraph on each.
Dermatology, Prescribing and Palliative Care Tutorials
Package Overview
Target Audience
This package is intended for 4th and final year medical students. “Prescribing in
General Practice” and “An Introduction to Palliative Care” may also be of interest
to post-graduate students and General Practitioners.
Purpose
This package consists of three modules “Dermatology”, “Prescribing in General
Practice” and “An Introduction to Palliative Care”. The first module introduces
students to the diagnosis and management of common skin conditions seen in
GP. Students are provided with tuition, practice and assessment on prescribing
and in the third module, students are introduced to the concept of palliative care
and palliative care services.
Content
Dermatology A tutorial approach has been adopted to present the characteristic
features, known causes, epidemiology and management options for the following
conditions: psoriasis, eczema and viral infections. Images are used to illustrate
and reinforce teaching points. Three cases are presented to enable the students
to assess their ability to diagnose and mange these conditions. A set of revision
questions are also included to enable them to assess their knowledge of the
material presented.
10
Prescribing in General Practice The module begins with an introduction to
prescribing, considering the reasons for prescribing and the issues that should be
considered before prescribing any drug. General guidelines which aim to ensure
more precise prescribing are outlined. Chapter 3, Writing a Prescription Form,
explains the prescription form and how it should be completed. Chapter 4,
Controlled Drugs, considers the guidelines for prescribing a controlled drug,
Chapters 5 and 6 focus on the issues and guidelines relevant to prescribing for
specific groups in the population, namely, children (chapter 5) and the elderly
(chapter 6). Chapter 7, Drug Interactions, identifies and briefly explores two types
of interactions, namely, pharmacodynamic and pharmacokenetic. Chapter 8,
Repeat Prescribing, explores the advantages and disadvantages of repeat
prescribing. Five systems of repeat prescribing are outlined and the criteria for a
good repeat system presented. Finally, a number of case studies are presented
which provide students with practice in completing on-screen prescription forms.
A drug database is also available for consultation.
Computerised Modified Essay Questions
Package Overview
Target Audience
“CMEQs’ is targeted at fourth and final year medical students. It may also
be of use to general practitioners in their continuing medical education
(CME).
Purpose
The package is based on the Modified Essay Question (MEQ) - a paperbased assessment tool which is used in undergraduate and post graduate
medical education world wide. It is intended to provide medical students
with a self-assessment tool to enable them to assess their ability to define
“problem lists” for patients and plan effective management strategies. A
wide range of skills can be tested. Emphasis is place on:




breadth of knowledge,
attitudes,
problem solving skills and
patient management.
11
Content
A series of clinical scenarios, based either in a doctor’s surgery or the
patient’s home, are presented. With each scenario the student is asked a
number of questions relating to how a particular set of problems should or
could be managed. The student answers the questions, either in point or
essay form, compares their answers with model answers and marks
accordingly. A number of reports are generated providing the student with
a profile of their performance on each question and a comparison of their
performance with that of their peers. Supplementary information is also
provided to enhance the learning value of the CMEQ.
12
PRACTICE BRIEFING
Please contact your practice Tutor at least two weeks before your attachment starts.
If you are staying in accommodation you will need to check arrangements for getting the
key.
If you do not have transport and the accommodation is far from the practice the GP tutor
will either make arrangements for you to be collected or else advise you on the local
public transport arrangements.
Please make sure you are punctual, and also if you are going to be absent from any
session please let the GP know in advance. This is a compulsory attachment so if you
are absent for several sessions you may be required to do an additional elective in
General Practice.
Your accommodation may mean you are on your own so it is suggested that you bring
an alarm clock and transistor radio. If you are staying in B&B accommodation ask if you
can use the microwave to heat food etc and bring a supply of coffee and biscuits with
you.
END COURSE ASSESSMENT
This consists of:
(1)
(2)
(3)
A short oral examination
A modified essay question paper
Objective Structured Clinical Examination (OSCE)
ORAL EXAMINATION
During this session your record card is examined and the assessment by your GP Tutor
is considered. You will also be asked about a common clinical condition and about
practice management.
MOBILE PHONES AND ELECTRONIC TEXT MESSAGING DEVICES
1. Mobile telephones and electronic messaging devices must be switched off during ALL
lectures, tutorials, practical classes and clinical teaching sessions.
2. Students must not bring mobile telephones or electronic messaging devices into
examination venues. This requirement applies to all written, practical and clinical
examinations as well as class tests. The rule extends to the time during some
practical and clinical examinations when a number of students have completed the
examination while others are waiting to take it. Students who do not comply with this
requirement will be subject to the University conduct regulations as outlined in
calendar Book One.
13
MODIFIED ESSAY QUESTION PAPER
This is an integral part of the Final MB Examination. It is also used in several
membership examinations.
At Queen’s the Modified Essay Questions are based on real cases submitted by Hospital
Consultants and General Practitioners. Generally speaking a clinical problem is
unfolded with the student being asked to comment at various stages on such items as
appropriate history and examination, diagnostic possibilities, relevant investigations,
ultimate diagnosis and various actions to be taken such as advice to the family,
mobilisation of relevant members of the Primary Care Team, and referral, if necessary,
to a specialist. The student will also be asked from time to time to prescribe medication
and suggest other possible sources of help.
The MEQ is not designed to test pure factual knowledge and therefore when setting and
marking the paper emphasis will be placed on how the student can apply clinical
behavioural and management skills to the various problems which are presented.
Although the word essay is included in the title this is a slight misnomer as most
questions are just answered as a series of headings and points. This saves you time
and also makes for easier marking by the examiners.
Concise and legible writing won’t necessarily gain any marks but conversely you
certainly won’t lose any!
Time is very important and is a major cause of students scoring low marks. Each
question scores separate marks and it is important to work out a time schedule and stick
rigidly to this. A candidate who only has time to answer 15 out of 20 questions will
struggle to achieve a pass mark no matter how well he/she has answered the questions
attempted.
It is also important to read each question carefully, for example if a question asks for a
list of physical signs of head injury, a skull X-ray will not receive a mark since this is an
investigation.
It is also advisable not to look through the paper before you start as this may distort your
answers at various stages which may be geared to the fact that certain information about
the case is being withheld to a later time.
If in doubt about answering a particular management problem try and think about exactly
what you would do, eg if starting a patient on antihypertensives, in addition to writing out
a prescription you would also counsel them about smoking, weight, diet, salt and lifestyle
etc. These would all be expected to be included in your answer and the marking
schedule would be structured accordingly.
Sometimes each page of the MEQ is marked independently - you should therefore
answer each question specifically even if this answer involves repetition of part of an
earlier section.
14
The modified essay paper is in many ways unique but provided the student has a sound
basis of clinical knowledge, common-sense and has brushed up on exam technique, this
type of problem solving exercise should be looked upon as a useful learning tool and not
too difficult a hurdle at Final MB. Some Specimen Modified Essay Questions are
included at the end of this Study Guide.
OBJECTIVE STRUCTURED CLINICAL EXAMINATION
This is part of your assessment and is run jointly with Geriatric Medicine. There are
stations covering Ethics, Clinical Skills, Communication Skills, Diagnostic Skills,
Prescription Writing and Data Processing.
A number of the stations have real or simulated patients to test clinical and
communication skills. Students spend 5 minutes at each.
Each station is marked using a structured marking sheet. This assessment enables
each student to be tested across a spectrum of skills in the different settings of general
practice and geriatric medicine.
GP ASSESSMENT
MEQ
30%
Oral
10%
GP OSCE Stations
60%
____
100%
____
The OSCE Stations have been standardised so the Pass mark varies between
Stations. The overall mark is re-adjusted to give a pass mark of 50%
15
Students who fail will have an interview and may be asked to repeat part or all of the
assessment.
Students who perform badly in all areas may be asked to resit the entire examination in
September prior to entering final year.
The aim of this assessment is to enable students to see for themselves the level of
knowledge and skills they have obtained during the course and compare themselves to
their peers.
FOURTH YEAR RECORD CARD
This record card acts as an aide memoir highlighting the learning opportunities which
exist during your time in the Department of General Practice and on your attachment to a
practice.
It contains suggestions for what to do on your attachment. We advise you to show this
to your GP teacher and put together a plan for your time when in the practice.
This year you are being asked to complete a mini CV and questionnaire to show to your
GP Tutor at the start of your attachment (see page 26).
It is important to get as much “hands on” experience as possible, eg interviewing
patients, prescription writing and doing practical procedures such as venepuncture. For
example, if you are spending time with the receptionist try to be “doing” rather than
“observing”, eg answering the phone, dealing with queries, pulling charts.
We do not expect you to do everything that is listed in the record card but it acts as a
guide for you and your tutor.
The record card is looked at in your oral assessment and the report from your GP tutor is
also taken into account.
EVALUATION AND FEEDBACK
This is a new course and much of it is in small groups. The more you participate the
more you will get out of it. We hope to get to know you well over the eight-week block. If
you have any queries please speak to Miss Cathleen Agnew, Telephone No. 028 90
204252. We will ask you to complete a questionnaire at the end of your attachment to
enable us to evaluate the course. This is anonymous. Dr Drew Gilliland is the Course
Co-ordinator and he would be very pleased to deal with any queries you have about the
course or other related matters. He is based in Room 8, 4th Floor, Dunluce Health
Centre, Telephone No. 02890 204300, E-mail: [email protected].
There will also be informal discussion about this course after the OSCE Feedback
session on Friday afternoon at the end of the 8-week block.
16
ATTENDANCE
This is a Clinical Clerkship and attendance is compulsory. A roll-call is taken at the
sessions in the Department of General Practice and the GPs are asked to monitor
attendance at practice level. The minimum attendance required is usually at least
90%. Students who miss more than this will be asked to provide an explanation. If you
have been ill a Doctor’s Certificate may be required.
If you have to miss a teaching session please contact us at 028 90204252 and let us
know.
You have a half day off on Wednesday afternoon in Weeks 1 & 5 and this time should be
used for personal matters such as shopping, personal banking etc.
STUDENTS IN DIFFICULTY
The Course in Primary Health Care is quite demanding. Students have a lot of
information and skills to learn. They are also asked to apply what they have learnt in
dealing with patients, relations and staff in the General Practice setting. Evaluations tell
us that students generally enjoy their time in General Practice and think it worthwhile.
However, we are aware that some students, for a variety of reasons, may experience
difficulty. They may have personal or health problems. If so, they should consider
speaking to their faculty tutor or a doctor in Student Health.
Alternatively please feel free to contact Dr Drew Gilliland or any of the other teaching
staff. Health related issues are best managed by your own Doctor but we would be
happy to give some initial advice. Also if you have had a recent life event such as a
death in the family it may be helpful for you to let us know. The information sheet on
page 17 gives specific information regarding potential sources of help.
Intimate Examination of patients by students
The Committee, under the chairmanship, of Professor Neil McClure looked at
this area in detail. This Committee has decided that vaginal examination can
only be performed by a medical student under the direct supervision of a
Gynaecologist. This has implications for General Practice and in the future
medical students are no longer permitted to perform vaginal examinations
whilst on their attachments in General Practice but are still able to observe
vaginal examinations carried out by other healthcare professionals provided
the patient has given their consent.
17
STUDENT SUPPORT AND GUIDANCE
INFORMATION SHEET
GENERAL INFORMATION
1.
Head of Student Support
Miss G. Silvestri, Head of Student Support and Guidance
Contact: 028 9063 3152 (answering machine available outside main
office hours)
2.
School Tutor
If you do not know who your Tutor is or how to contact them speak to
Mrs Perpetua Lewis, Department of Ophthalmology
Contact: 028 9063 3152 or email: [email protected]
3.
The web
Student Support and Guidance Website: http://www.qub.ac.uk/fmhs/ssg/
Students News site: http://www.qub.ac.uk/fmhs/news.html
Information on Facilities in Hospitals and who to contact for help while on
attachment: http://www.qub.ac.uk/fmhs/sumde/Index.htm
4.
School Office
School Manager: Mrs K Copeland
[email protected], Tel 028 9097 3258
Curricula enquiries: Mrs Linda McGuinness
[email protected], Tel: 028 9097 1448
Exam Enquiries: Mrs Arlene Stockman
[email protected], Tel: 028 9097 5963
Student Support: Mrs Nicola Swenarton
[email protected], 028 9097 1451
5.
Female Medical Student Counsellor
Dr Ann Harper: 028 90263009 or 028 90240503 Ext 2506
SPECIFIC CONTACTS FOR THIS MODULE/COURSE
Dr AEW Gilliland – Course Co-ordinator
tel: 028 9020 4252; fax: 028 9031 0202; email: [email protected]
18
This section, pages 18 - 26, will be handed in at the Oral Station of the
Objective Structured Clinical Examination (OSCE). These pages must be
stapled before being handed in.
DEPARTMENT OF GENERAL PRACTICE
QUEEN’S UNIVERSITY OF BELFAST
FOURTH YEAR CLERKSHIP 2006-2007 RECORD CARD
NAME _______________________
19
ACTIVITIES
A.
1.
2.
3.
4.
5.
6.
7.
8.
Sitting in on surgery consultations.
Accompanying GP on home visits.
Sitting in with another G.P.
Spending a night “on call”.
Visiting the local chemist.
Writing up notes.
“Out of Practice” eg Case Conference, Court, Inquest.
Accompany GP to visit a patient with a terminal illness.
B.
Sessions with other primary care team members.
Staff
Details of Session
Receptionist (s)
Health Visitor
District Nurse
Treatment Room/Practice Nurse
Community Psychiatric Nurse
Social Worker (if attached to practice)
Practice Manager
C.
Tutorials with GP Tutor
1.
2.
Common Paediatric Problems
Gynaecology
List any other Tutorials
1.
2.
3.
4.
20
Please ask your GP teacher to initial those activities you feel you have carried out
adequately. These are only suggestions. You do not have to do them all and you
may add more at the bottom.
Task
Interviewing 5 or more patients on your own.
Writing a hospital referral letter on your own.
Presenting a case report of a patient you have
interviewed at length at home.
Familiarisation with appointments, house calls and repeat
prescribing systems of the practice.
Writing prescriptions.
Familiarisation with practice computer system.
Treatment Room Procedures
BP measurements
Urine analysis
Ear syringing
Dressing wounds
Suturing
Removing Sutures
Taking blood
Giving injections
Observing cervical smears
Follow up of an acute admission.
Initial
21
PRACTICE ATTACHMENT
Please complete this timetable together with your GP teacher so that time available
can be used as efficiently as possible. Opposite are some suggested activities.
Week 2/6
AM
Mon
Tues
Wed
Thurs
Fri
PM
RETURN TO DEPARTMENT OF GENERAL PRACTICE
Week 3/7
AM
PM
AM
PM
Mon
Tues
Wed
Thurs
Fri
Week 4/8
Mon
Tues
Wed
Thurs
Fri
Please asterisk any evening when you were “on call”.
Students finishing their Clerkship in Week 4 stop Friday lunchtime. Those finishing
in Week 8 stop Thursday pm. The combined Primary Health Care and Ageing &
Health Assessment is on Friday morning of Week 8 at 8.45am. A more detailed
Programme will be circulated to you before you go out on your 2nd clinical
attachment during the 8-week Block.
22
To the G.P. Teacher:
Please complete this form with your student. This is a means of giving the student
some valuable feedback.
Student’s Name
_____________________________________
Date of Attachment _______________ to ____________________
Poor
Attendance
Punctuality
Enthusiasm
Communication Skills
Clinical Skills
Medical Knowledge
1
1
1
1
1
1
Excellent
2
2
2
2
2
2
3
3
3
3
3
3
4
4
4
4
4
4
5
5
5
5
5
5
Please comment on areas that are very good and those that could be improved
upon.
GP Sig. __________________________________
Surgery
Stamp
23
PRACTICE EXPERIENCE QUESTIONNAIRE
1.
Name of Student
2.
Name of Tutor
3.
Address of attached practice
4.
List size
5.
Percentage of patients
Under 5
6.

65-74

Over 75

Practice features (please circle)
Health Centre
Partnership owned
Urban
Rural
Mixed
Dispensing
7.
Please comment on any special, social or other features of the
practice.
8.
Is Child Health Surveillance carried out?
YES/NO
9.
Is minor surgery carried out?
YES/NO
10.
What Health Promotion clinics are carried out?
11.
What additional clinics/sessions exist within the practice, eg
obstetrics, immunisations, police work, factory work?
24
12.
What systems of audit exist in the practice?
13.
What staff are employed/attached:
Nursing/Paramedical:
1.
2.
3.
4.
5.
Clerical/Administrative: 1.
2.
3.
14.
What special diagnostic, therapeutic or other equipment is available
within the practice?
1.
2.
3.
4.
15.
What diagnostic facilities are available by direct access
outside the practice?
Diagnostic
1.
2.
3.
4.
16.
Is the practice computerised?
If yes, what are its principal uses?
1.
2.
3.
4.
17.
How are out of hours duties covered?
YES/NO
25
18.
Please record number of patients seen during the second week of your
attachment.
M
T
W
T
F
With your GP Tutor
Interviewed initially by yourself
Seen during Special clinics
Home visits
Night calls (after 6 pm)
TOTAL
19.
STUDENT’S OWN IDEAS AND LEARNING EXPERIENCE
Describe two features (50 words each) of the practice which you have found
interesting or stimulating or which have influenced your attitude or helped
improve your knowledge and skills.
20.
What changes (if any) would you like the practice to make for future
student attachments? Your comments will be handled sensitively by the
Department of General Practice.
26
21.
How do you rate the educational experience you have had during your
practice attachment (please circle)?
1
Very good
22.
2
Good
3
Average
4
Below average
5
Poor
CLINICAL DIARY
Please list 10 cases interviewed by you personally during the course of your
attachment. You may be asked about some of these in your oral examination.
Date
(1)
(2)
(3)
(4)
(5)
(6)
(7)
(8)
(9)
(10)
Patients Initials
Age
Sex
Reason for Consultation
27
MINI CURRICULUM VITAE
Name: ______________________________
Mobile Telephone Number (if applicable):
____________________
Town or City where you grew up:
____________________
Outside interests and Hobbies:
________________________________________________________________
________________________________________________________________
________________________________________________________________
Proposed Elective Destination:
_____________________________________
Any special requirements during attachment:
________________________________________________________________
________________________________________________________________
________________________________________________________________
________________________________________________________________
________________________________________________________________
________________________________________________________________
28
SELF ASSESSMENT QUESTIONNAIRE
This self-assessment is to assist students and GP Tutors in order to make the
Clerkship in General Practice as relevant and useful as possible for each student.
Please complete this Questionnaire which is attached to the front of this
Study Guide at the beginning of your attachment and show it to your Tutor.
Keep this for reference during your attachment. Please bring this
Questionnaire with you on your End Course Assessment day.
Obviously it is not possible to cover the list of topics in a 3 week Clerkship but it will
give the student and tutor a guide to areas which might be covered. Students at
the beginning of the year will have less confidence in many areas but as the year
progresses experience and teaching in other 4th year modules should mean that a
number of the topics have already been covered in some depth.
You will be given a similar Questionnaire at the end of your Attachment.
You will have a code number assigned so we can analyse the difference between
the two but no individual student will be identified.
This is a pilot and we will be interested in feedback from students and GP Tutors.
29
WHAT IS GENERAL PRACTICE
This section gives some background information about General Practice. It will
describe the role of the GP and members of the Primary Health Care Team and
will outline the role and content of General Practice.
The definition of a general practitioner by The Royal College of General
Practitioners in 1972 is still useful.
“The doctor who promotes personal, primary and continuing medical care to
individuals and families. He* may attend his patients in their homes, in his
consulting room or sometimes in hospital. He accepts the responsibility for making
an initial decision on every problem his patient may present to him; consulting with
specialists when he thinks it appropriate to do so. He will usually work in a group
with other general practitioners, from premises which are built or modified for a
purpose, with the help of paramedical colleagues, adequate secretarial staff and all
the equipment which is necessary. Even if he is in single-handed practice he will
work in a team and delegate when necessary. His diagnosis will be composed in
physical, psychological and social terms. He will intervene educationally,
preventatively and therapeutically to promote his patients’ health”.
Because this role includes psychological and social diagnoses he has to have a
much wider information base than previously. By including prevention and
education he extends his interest to people who do not call on his services
spontaneously and who are not necessarily ill at all.
This wider role of the general practitioner places him firmly in the context of
Primary Health Care which is the more appropriate term to cover the whole of the
team.
General practice, by tradition, was mainly concerned with treatment of disease
presented by patients, while the emphasis on primary care is on health and its
promotion by a healthy life style, by prevention of ill health if possible and by the
provision of acceptable first line health care to which all people have access and
which they can afford.
Factors changing within general practice
1.
2.
3.
4.
5.
6.
7.
8.
9.
Increasing interest in performance review (audit)
Increasing interest in prevention of ill health
Increasing application of computers and information technology with a
reduction in information transmitted and held on paper record
Working in teams
More management education for GPs and staff
More practice managers
Increasing use of district and treatment room nurses
Social-Skills training in consultations
Patient Participation groups
* He/She is interchangeable
30
10.
11.
12.
13.
14.
More female general practitioners
Increasing/changing workload
More effective treatments available in general practice
Consultant led clinics in general practice
Near patient testing eg blood investigations
Factors for change from outside general practice
1.
2.
3.
4.
5.
6.
7.
Financial cuts in the NHS
Few hospital beds
Shorter hospital stay
Day Case Surgery
Emphasis on Care in the Community
Longer waiting lists in certain areas
Super specialisation
Social changes
1.
Population changes. In the past 20 years the number of people aged more
than 65 years has increased by 1/3. They now constitute 15% of the
population and the proportion of elderly is set to rise further. There is also an
increasing Ethnic Minorities population. There is increased unemployment,
more work-related stress and a tendency to stop work earlier.
2.
Technical advances in coping with disability.
PRIMARY AND SECONDARY CARE
One of the major differences between primary and secondary care is the nature of
presentation of illness. In secondary care patients are usually referred from
another practitioner, their illness has been categorised, and by the very nature of
their presentation in hospital, social and psychological causes for their illness have
usually been excluded. In primary care patients present to both nurses and
doctors with largely unsorted problems.
Primary care is not the sum of a range of secondary care specialities performed to
a lesser degree, it has knowledge and skills peculiar to it. These skills relate to the
work of a clinical generalist, and focus around problem definition and solving. A
clinical generalist is able to understand the illness, the patient and the context,
bringing together physical, psychological and social causes of disease.
Generalists use their skills in the consultation, both in communication and
examination rather than advanced technology to identify the nature of disease.
Investigations are kept to a minimum and tend to follow a progression rather than a
battery as seems to occur in secondary care. Perhaps the most important feature
of the clinical generalist is their ability to use a hypothetico-deductive model to
identify disease. Using their knowledge of the patient, their social context,
including their working environment and home surroundings, the
31
common illnesses in the community and their wider knowledge of disease, a
generalist will identify a priority list of likely causes for a patient’s symptoms. He
will then ask discriminating questions to exclude or include possibilities from the
list. Although this process may occur from start to finish in one consultation, more
commonly the generalist allows the full picture to unfold over a series of planned
consultations. The use of information collected over time is a special feature of
general practice in the United Kingdom and enhances our generalist role.
An example might be a child who presents with a cough. A child whose relatives
have asthma, and who has been seen with hay fever or eczema would make a
diagnosis of asthma likely; discriminating questions might focus on the effect of
exercise or winter viruses. Examination might concentrate on listening for wheeze
or checking the peak-flow rate. A child of the same age who had recently returned
from visiting grandparents on the Indian subcontinent would raise the possibility of
tuberculosis. Discriminating questions might focus around fever and weight loss.
A chest x-ray would be a high priority. A child of the same age who appeared well,
perhaps even disruptive in surgery and whose mother had a previous history of
postnatal depression might be the presenting symptom of a maternal depressive
illness. Questions in the last example may need to move from inquiring about the
child’s health to asking about the mother.
A clinical generalist is usually skilled at picking up small verbal and non-verbal
cues, they will have a range of skills which will help them reach the most likely
diagnosis quickly, excluding serious illness and creating a safety net if their original
hypothesis is incorrect. They also have skills in sharing with the patient their
thinking and their proposed management, listening and asking for the patient’s
ideas and anxieties about their illness. These skills help the patient to understand
the nature of their illness which increases the cost-effectiveness of any treatment.
Clinical generalists also have to anticipate and deal with acute life-threatening
illness: the child with the cough might present acutely, either during the day or at
night, severely ill and at risk of respiratory arrest. They also need a range of skills
to manage health promotion and disease prevention, perhaps drawing a parent’s
attention to the effect that a smoky atmosphere might have on their child’s cough,
or being aware that the child with a cough has not been immunised against
diphtheria.
Managing illness in primary care has its own special skills. Patients have total
autonomy, once they have left the consulting room they are free to follow advice or
ignore it. There is evidence to suggest that about one third of prescriptions are not
presented to the chemist. Successful management of disease requires a
negotiated plan in which the patient has ownership and understanding; it will
usually involve pragmatic compromise over a theoretical academic clinical ideal in
favour of what is feasible in the patient’s social context and understanding. Each
management plan in primary care must address the three aspects of illness: the
disease, the patient and the social context.
32
What is a team?
A team is defined as a group of people who make different contributions towards
the achievement of a common goal.
Gilmore et al (1974) described four essential characteristics of teamwork in general
practice.
1.
The members of the team share a common purpose which builds them
together and guides their outcomes.
2.
Each member of the team has a clear understanding of his/her own
functions and recognises common interests.
3.
The team works by pooling knowledge, skills and resources and all
members share responsibility for outcome.
4.
The effectiveness of the team is related to its capabilities to carry out its
work and its ability to manage itself as an independent group of people.
The Primary Health Care Team is an organised group of professional workers,
each of whom is directly available to people in a defined community.
The Primary Health Care Team
1.
2.
3.
4.
5.
6.
7.
Permits division of labour and prevents dysfunction of work.
Improves interdisciplinary understanding.
Extends individuals’ capabilities.
Makes communication easier.
Provides a basis for the appraisal of individuals’ performance.
Allows specialisation.
Permits changes to be made.
Those who make up a Primary Health Care Team vary but usually include the
General Practitioner, Health Visitor, District Nurse, Practice/Treatment Room
Nurse, Social Worker, Receptionist and Practice Manager.
The Health Visitor
The Health Visitor is a state registered nurse with a post-registration qualification
who provides a continuing service to families and individuals in the community.
He/she has a broad potential brief which includes:1.
Postnatal home visits
This is a statutory duty from the day the midwife stops attending (usually
day 10).
33
2.
*
Regular follow-up at home or surgery continues until the child attends
school. There are routine assessments at 6 weeks, 7 months, 18 months,
30 months and pre-school at 4 years. The function is to educate and
support the mother in basic baby care, common postnatal problems, minor
childhood illness and developmental milestones. The visits also include
child developmental and screening work including tests of sight, hearing and
development.
Specific surveillance and support of at risk groups
single parent families
cases of potential non-accidental injury
the handling of children with emotional and behavioural problems or
physical and mental handicap
marital counselling
4.
*
*
*
Preventative work and Health Education
immunisations
health education
visits/screening of elderly
5.
*
Advisory and liaison work
advice on local resources eg mother and toddler groups, day nurseries,
self help groups.
3.
*
*
*
The District Nurse
Broad potential overlaps with other members of the team but main duties are:1.
*
*
*
*
*
*
*
*
*
*
2.
General nursing care including
prevention of pressures sores
bowel and catheter care
treatments, injections and dressings
post op assessment and care of patients discharged early from hospital
venepuncture
rehabilitation
Stoma care
management of incontinence
care of patients with diabetes
care of the terminally ill including the use of a syringe driver to provide
symptom relief
Mobilisation of resources, incontinence aids, commodes, ripple beds, bath
aids. The district nurse co-ordinates the nursing care of the terminally ill and
involves the Twilight Nursing Service, the Marie Curie Nursing Service and
help from the local hospice whenever appropriate.
34
3.
*
*
*
Preventative work
monitoring at risk groups
health advice
administration of influenza and pneumococcal inoculations.
The Treatment Room Nurse/Practice Nurse
The role of this nurse depends in part on the personal interests, confidence and
experience of the nurse. It is also depends on the needs and requirements of the
Practice.
A Practice Nurse may share district nursing duties. Commonly performed tasks
include:a.
Basic nursing procedures
dressings, venepuncture, injections, immunisations, basic observations
(weight, BP, urine testing), suture removal, ear syringing, taking swabs for
culture and sensitivity and recording ECGs.
b.
Management of minor accidents.
c.
Special Clinics - Wart Clinics (application of liquid nitrogen), Diabetic Clinics,
Hypertensive and Well Woman/ Well Man Clinics.
d.
Taking cervical smears (if trained to do so)
e.
Organising the monitoring of patients on warfarin and cytotoxic agents
f.
Assisting the doctor in various procedures (eg fitting of intrauterine
contraceptive devices (I.U.C.D.s), minor surgery) and acting as a chaperone
g.
Regulation and maintenance of treatment room supplies and equipment
h.
Completion of paper work for audit and items of service payments.
i.
General health education and advice.
The Community Psychiatric Nurse
Community Psychiatric Nurses (CPN) are usually based in a Health Centre or
premises belonging to the Community Trust. They provide care for people
suffering from severe and chronic mental illness and for those suffering from less
severe but nevertheless disabling mental health problems.
35
The care provided includes assessment of the mental state, monitoring the mental
state, administration and monitoring of medication including compliance, supportive
psychotherapy and rehabilitation.
They also provide support, advocacy, information and advice to sufferers’ relatives
and carers.
User and Carer participation is encouraged and valued.
The Reception Staff
They are key workers as they are the usual initial contact for access to any service
offered by the Practice. 40% of their workload involves patient contact, face to
face or by phone.
Reception duties
*
*
*
making new and repeat appointments
receiving and directing patients
taking requests for visits.
Filing and record duties
*
*
*
*
*
*
locating patients’ records and re-filing
dealing with post
registering new patients
filling in claim forms
updating clinical records
administering the repeat prescribing system.
*
secretarial duties including typing letters, organising clinics and patient
recall.
The Social Worker
Some practices have attached Social Workers, others do not, so the student may
not meet a social worker during their practice attachment.
Their work load covers three broad areas
A.
*
*
*
Individual case work
counselling individuals and families with financial and personal problems
marital and bereavement counselling
follow up and support for the mentally ill.
36
B.
Advice and allocation of resources.
*
*
home help, meals-on-wheels
social service day centre places
*
advice to impoverished, disabled and homeless, home adaptations,
telephone
installation grants, social benefit allowance, legal housing rights,
voluntary and self-help groups.
C.
Statutory responsibilities*
*
*
*
*
*
supervision of children in Care
supervision of adoption, fostering and child minding
management of child abuse care
compulsory admission of patient under the Mental Health Act 1983
responsibility for the handicapped under the Disabled Persons Act 1970.
Any one social worker could not possibly deal with all of the above. Duties such as
the supervision of children in Care is specialised and requires additional training.
However, a practice based social worker can be the first point of contact. If s/he
cannot deal with this work it may be referred appropriately to another social worker.
*A practice based social worker would generally have less statutory responsibilities
to deal with.
Practice Manager
Most practices now employ a Practice Manager, even the smaller practices may
employ someone on a part-time basis.
Many of their duties can be delegated but they retain an overall responsibility for 6
broad categories.
1.
*
*
*
*
*
Staff
hiring and firing
induction and training
rotas and holidays
contracts
grievances and problems
2.
*
*
*
*
*
Finance
monitoring and maximising sources of income
monitoring all outgoings
paying staff salaries
record keeping
accounts preparation
37
3.
Administration, organisation of basic administration tasks.
4.
Premises and supplies, organisation of security, equipment maintenance.
5.
Future planning.
6.
*
*
Liaison work
staff, doctors, hospitals, Health Boards
organisation of meetings.
Additional Reading, pages 1 -24 “Clinical Method”, Fraser. 3rd Edition.
Pages 1 - 67, “Notes for the MRCGP”, 3rd Edition, Palmer.
THE NEW GENERAL PRACTICE CONTRACT
This contract was introduced in April 2004 and replaces the previous contract
which was introduced in 1990. From a patient’s perspective, the two major
differences are that patients are no longer registered with an individual GP but with
a practice and practices can now opt out of providing out-of-hours cover. Out-ofhours provision will now be organised by Health and Social Services Boards in
Northern Ireland and Primary Care Trusts in England, Scotland and Wales.
There are a number of clinical domains where the quality of care the practice
delivers is looked at and individual practice incomes are adjusted accordingly.
CLINICAL DOMAINS
Heart failure
Palliative Care
Dementia
Depression
Chronic Kidney Disease
Atrial Fibrillation
Obesity
Learning Disabilities
Smoking
Coronary Heart Disease
38
Left Ventricular Dysfunction
Stroke and TIA
Hypertension
Diabetes mellitus
COPD
Epilepsy
Hypothyroidism
Cancer
Mental Health
Asthma
Practices can also decide to perform additional services called “Enhanced
Services”. This involves the practice developing specific programmes to look at,
for example, alcohol misuse, anti-coagulation monitoring, IUCD fitting, drug
misuse, intermediate care, care of the homeless, sexual health services and minor
surgery. Practices also receive support for providing premises and the provision of
information technology.
You will not be expected to know details about the Contract but this basic
information should be useful to you when you are attached to your Practice.
39
CONSULTATION SKILLS
(WATCHING YOU, WATCHING ME)
Sessions during Course
Week 1
Mon 2.00pm
Dept. of General Practice
Week 1 or 5
Thurs 2.00pm
Dept. of General Practice
Week 1 or 5
Fri 2.00pm
Teaching Practices
Week 2 or 6
Fri 2.00pm
Teaching Practices
AIM:
How to develop an effective consulting style.
OBJECTIVE:
1.
To demonstrate ineffective consultations.
2.
To provide evidence that quality consultations can be related to positive
health outcomes.
3.
To describe health belief and task based models of the consultation.
4.
To present a series of five tasks which can be used to practice and teach
effective consulting.
5.
To demonstrate how students can identify their strengths and weaknesses
by videotaping their own consultations.
6.
To enable students to practice the use of recommended techniques for
discussing videod consultations in terms of evaluating them and giving and
receiving feedback.
CONTENT
One introductory tutorial followed by one practical using simulated patients.
Two CCTV sessions when students will consult with both patients.
TASKS
1.
How would you define clinical skills, communication skills and consultation
skills.
2.
What factors make a successful consultation.
3.
What are (a) the doctor (b) the patient barriers to effective consultations.
PRESENTATIONS
1.
Evidence for poor consultations.
2.
Evidence that consultations can be effective.
3.
Clinical Interviewing Skills.
4.
Consultation Models.
5.
Performance Tasks.
6.
Discussing video consultations.
40
CONSENT
It’s a general legal and ethical principle that valid consent must be obtained before
starting treatment or physical investigation or providing personal care for patients.
This principle reflects the right of patients to determine what happens to their own
bodies and is a fundamental part of good practice. A health professional who does
not respect this principle may be liable both to legal action by the patient and
action by their professional body. With regard to medical students written consent
is not normally required for a patient either to sit in on a consultation with a GP and
patient or interview and/or examine the patient themselves. However whenever
possible patients should be informed at the time of booking the consultation that
the medical student may be present. If this is not possible they should be told when
they book in with the receptionist and should be given the choice of asking the
student to wait outside. It is a sensitive area and GPs and students need to tread a
fine balance between getting the student exposed to a range of important medical
conditions during their training but at the same time ensuring patients do not feel
compelled to take part in consultations or procedures involving a medical student.
Intimate examinations involving examinations of the female breasts, the genitalia
and the male and female rectal areas should only be observed or carried out by a
student under supervision by a general practitioner or health professional with a
chaperone if needed (see medical school guidelines). A student must never
perform any of these examinations on his or her own or without consulting the
health professional supervising the teaching session.
CONSENT TO VIDEO RECORDING.
There are two video recorded teaching sessions in the fourth year general practice
course and patients are booked for a video recording consultation with a fourth
year medical student with the general practitioner in attendance. Box 1 sets out the
GMC basic principles for making and using visual and audio recordings of patients
(2002) and Box 2 shows the patient consent form for video recordings which is
used for fourth year teaching. It is important in this process that patients are aware
beforehand that the consultation will be recorded by video camera by the student
undertaking the interview. The GP and other students will be watching either
through a one-way mirror or using a television monitor. Patients are aware that
they have the right for this tape not to be subsequently used in a teaching session.
They are also told that the tape is wiped at the end of the teaching session after
being played back to the student, the other students in the teaching session and
the teacher. If the recording is to be kept this would require more detailed written
consent such as the RCGP consent form and the medical student would also have
to give their written consent as well.
41
Box 1 GMC basic principles, taken from Making and Using Visual and Audio
Recordings of Patients, 2002
________________________________________________________________
1 When making recordings you must take particular care to respect patients’ autonomy
and privacy, since individuals may be identifiable to those who know them from minor
details that you may overlook. The following general principles apply to most
recordings (exceptions are explained in the GMC guidance).








Seek permission to make the recording, and obtain consent for any use or disclosure.
Give the patient adequate information about the purpose of the recording when
seeking their permission.
Ensure that the patient is under no pressure to give their permission for the recording
to be made.
Stop the recording if the patient asks you to do so, or if the recording is having an
adverse effect on the consultation or treatment.
Do not participate in any recording that is made against a patient’s wishes.
Ensure that the recording does not compromise the patient’s privacy and dignity.
Do not use recordings for purposes outside the scope of the original consent for use,
without obtaining further consent.
Make appropriate secure arrangements for the storage of recordings.
2 If children who lack the understanding to give their permission are to be recorded, you
must obtain permission to record them from a parent or guardian. Children under 16
years of age who have the capacity and understanding to give permission for a
recording to be made may do so. You should make a note of the factors that were
taken into account in assessing the child’s capacity.
3 If a mental disability or mental or physical illness prevents the patient from giving their
permission, you must obtain agreement to recording from a close relative or carer. In
Scotland you must seek agreement from any person appointed under the Adults with
Incapacity (Scotland) Act 2000 who has an interest in the welfare of the patient.
4 People who agree to recordings being made on behalf of others must be given the
same rights and information as patients acting on their own behalf.
42
Box 2
Department of General Practice – Queen’s University Belfast
Patient Consent to Video Recording for
Undergraduate Medical Teaching
Date …………………….
Patient’s name ……………………………………………………………………….



Dr ………………………………., whom you are seeing today is hoping to video
4th year undergraduate students consulting with patients. The purpose is to
enable students to improve their communication and clinical skills. The tape
will be played back to the student with 2-3 other students who are part of the
teaching session and will be interviewing other patients. The tape will be
erased at the end of this teaching session.
The video is ONLY of you and the doctor or medical student talking together.
The camera will be switched off at any time if you wish. All video recordings
are carried out according to guidelines issued by the General Medical Council.
You do not have to agree to your consultation with the doctor being recorded.
If you do not want your consultation to be recorded, please tell the doctor. This
is not a problem, and will not affect your consultation in any way.
To be completed by the Patient
I have read and understood the above information and give my permission for my
consultation to be video recorded.
……………………………………….
Signature of the patient
BEFORE THE CONSULTATION
Date ………………
After seeing the doctor I am still willing for the recording of this consultation
to be used in this teaching session.
……………………………………….
Signature of the patient
AFTER THE CONSULTATION
Date ………………
To be filed in the Patient’s Notes
43
COMMUNICATION IN MEDICINE
An Age-Old and Essential Component of Medical Care
“I observe the Physician, with the same diligence, as he the disease; I see he
fears, and I fear with him: I overtake him, I overrun him in his fear, and I go
the faster, because he makes his pace slow; I fear the more, because he
would not have me see it. He knows that his fear shall not disorder the
practise and exercise of his Art, but he knows that my fear may disorder the
effect, and working of his practise.”
John Donne
1572-1631
CLINICAL INTERVIEWING SKILLS
INTRODUCTION
The interview process is a key component of all medical consultations. History
taking notes commonly list specific questions to be asked to identify common
symptom patterns and characteristics. It is essential to have mastered and
internalised these questions. However, a clinical interview is not the administration
of a questionnaire. There are basic skills which have applicability to all types of
interviews and are important irrespective of the nature of the patient’s complaints,
whether physical or psychological, and irrespective of the discipline of the
interviewer, whether social worker, general practitioner, psychiatrist, surgeon or
physician.
The basic skills will be described below but it should be remembered that such
skills are acquired and maintained by practice.
Diagnosis and treatment begin with the interview. Communication is about
gathering information as efficiently as possible in the time available and learning to
define as precisely as possible the patient’s problems and why he or she may have
chosen to come to the doctor. Knowledge and experience of communication must
be blended with knowledge of medicine and clinical reasoning to produce an
effective interview process.
INTERVIEWING STYLE
A.
BEGINNING THE INTERVIEW
Non verbal cues contributing to the development of a supportive atmosphere.
Introduction:
RELAX - not too formal
FACE PATIENT SQUARELY
44
EYE CONTACT
SMILE/FACIAL EXPRESSION
PROXIMITY - remove barriers if possible
TOUCHING PATIENTS - may be appropriate
TIME - always limited
B.
INTERVIEW PROCEDURES
Appropriate use of the following:
1.
2.
3.
4.
5.
6.
7.
8.
Listen carefully to the patient.
Attend to both verbal and non-verbal cues.
Facilitate using both verbal and non-verbal responses.
Control the interview by encouraging the patient to keep to the point.
Clarify by appropriate use of questions, repetition, summarising.
Avoid jargon.
Avoid premature focus on the initial problem.
Appropriate use of questions:
Use open questions before going on to ask specific questions.
Use single questions.
Avoid leading questions.
A positive and supportive atmosphere can be enhanced by informality of
style, a pleasant and warm manner, good eye contact, avoidance of
physical barriers, proximity.
C.
ENDING THE INTERVIEW
1.
2.
Summarise the problem.
Ask if there is anything else troubling the patient. Would he/she like to ask
any questions.
Empathy of manner and tone of voice are often significant
3.
COMMUNICATION SKILLS - A LIST OF BEHAVIOURS THAT CAN BE
OBSERVED IN EVERYBODY
THE CLINICAL INTERVIEW SETTING
A.
“SOCIAL” SKILLS
The beginning
Body posture
Eye contact
Attentive listening
Use of facilitation
Style of questions
45
Absence of jargon
Keeping patient to relevant matters
Picking up verbal cues
Presence of empathy
B.
“PROFESSIONAL” SKILLS
Use of confrontation
Use of silence
Appropriateness of interrupting
Picking up non-verbal cues
Covering psychological aspects
Covering personal issues
Covering social aspects
Ability to clarify
NOTE
In senior medical students and junior doctors the so-called “social” skills listed
above are likely to be used. Some training is required to ensure that these skills
are used effectively. Again in senior medical students and junior doctors the so
called “professional” skills are not likely to be effectively used and definite periods
of small group training are needed which should be repeated at regular intervals
over an extended period of time (several years).
46
EVIDENCE-BASED COMMUNICATION?
This session is written as a literature review, clustering around a number of subheadings. These sub-headings may themselves be used as the starting-point for
discussion, but participants may need to be persuaded to stay as close as possible
to the evidence.
Does doctor-patient communication need to be improved?
Complaints against doctors have increased dramatically since the introduction of
the health reforms, although there are many factors other than the doctor’s
consultation skills that may account for this (eg under-resourcing, consumerism,
political ideologies that increase public expectation beyond what the service can
deliver, the recession, a general culture of whinging, scape-goating and distrust of
authority). However, in 80% of cases, dissatisfaction with health care services is
deemed not due to clinical negligence but to some form of dysfunctional
communication eg inadequate information giving, misunderstandings, perceived
insensitivity, lack of control over decisions and treatment, discontinuity of care and
general disorganisation of the service (this, however, hinges on a definition of
communication so broad that it includes most things).
A 1995 study into information-giving prior to prostate surgery unearthed a clear
need to improve the comprehensiveness, relevance and intelligibility of information
given to patients.
There are a number of reviews of doctor/patient communication which are good
starting points for those with a serious interest in the topic. These include
Pendleton, the Toronto statement and Stewart.
Can communication be measured objectively? If so, what process and
outcome measures are chosen?
Studies of medical communication have been disappointingly restricted in their
approach. There are a few full-length qualitative studies eg Fisher and Todd (eds),
West, and Mishler - the last named is perhaps the best: all are similar in approach.
The great majority of the work undertaken, however is quantitative. A fair number
of these studies, it must be said, are scientifically elegant and linguistically naive,
and there is a disconcerting tendency for considerable resources to be elaborately
spent on the obvious (“Being nice to your patients is a good thing” - “Grass is
green’ - boffins’ amazing claim!”)
This at times is enough to make any rational humanists fall on their swords, but at
least it can be said that there is now a very considerable number of objective
studies which conclude that making the effort to communicate well makes a
difference.
Some process measures which have been used include: the time doctor and
patient spend talking and listening, the number of open questions asked, the
amount of information given, and the extent of opportunistic health promotion.
47
Outcome measures include: Patient factors eg satisfaction, psychological state,
recall and understanding of the consultation, symptomatic improvement,
quantifiable improvement in pathological state.
Doctor factors include: self-confidence, self-worth, job satisfaction, psychological
state.
What evidence is there that good communication improves outcome?
There is now some evidence that the quality of clinical communication is related to
positive health outcomes.
 Reduction in blood pressure is significantly greater in patients who, during visits
to the doctor, have been allowed to express their health concerns to the doctor
without interruptions.
 Concordance between doctor and patient in identifying the nature and
seriousness of the clinical problem is related to improving or resolving the
problem.
 Explaining and understanding patient concerns, even when they cannot be
resolved, results in a significant fall in anxiety.
 Greater participation by the patient in the encounter improves satisfaction and
compliance, and outcome of treatment (eg control of diabetes and
hypertension).
 The level of psychological distress in patients with serious illness is less when
they perceive themselves to have received adequate information.
 In surgery, beneficial clinical communication results in less pre-operative
anxiety, reduced post-operative complications and quicker and less stressful
recovery.
 In radiotherapy, beneficial clinical communication results in greater treatmentrelated knowledge and less emotional distress.
 Beneficial clinical communication results in increased compliance with medical
instructions and prescribed medications, leading to improved functioning in
everyday life, particularly for those with chronic illnesses.
 A recent systematic review of 10 analytical studies and 11 randomised
controlled trials indicate that, in 16 studies, more effective communication was
significantly related to improved health outcomes.
Can good communication be taught? Is it retained and used in real life?
Beneficial clinical communication can be taught, is feasible routinely in clinical
practice and can be achieved during normal clinical encounters, without unduly
48
prolonging them, provided that the clinician has learned (and is motivated to use)
the relevant techniques.
What are “the relevant techniques”?
These are communication skills which, when taught to and used appropriately by
doctors, have been shown to improve the outcome of consultations. They read like
a combination of common sense and good manners: but the techniques may not
always be applied in the heat of everyday practice.
These skills include such things as using a judicious mix of open and closed
questions; permitting silence to give the patient time to talk; checking
understanding; offering negotiated management; and also such things as ensuring
a quiet and comfortable environment for consultations to take place.
Such matters are fairly easy to teach and learn. Poor communicators, however,
are more likely to be poor because there is something wrong at the level of attitude
rather than skill. Problems, in other words, may be “deep” rather than “surface”. A
self-image which includes a sense of one’s own importance and the value of one’s
time can easily surface as brusqueness and indifference, for example. These
matters are more difficult to put right, and communication skills teaching must offer
the opportunity to bring out the attitudes which underpin performance in individual
consultations.
Evidence suggests that the effectiveness of communication teaching is much
enhanced if participants are able to reflect on and discuss a video/audio recording
of their performance. Feedback is essential to changing attitudes and behaviour.
This can also be powerfully provided by interacting with role players who, after the
consultation, can provide the doctor with accurate information about how s/he
made the patient feel, how competent and empathic s/he appeared to be, what
might be done to improve certain areas etc. This is perhaps the only time in a
doctor’s career that such detailed insight into his/her effect on patients will be
offered.
What aspects of communication do patients complain about?
The commonest complaints in both hospital medicine and general practice are:
 Time factors (too long waiting, too short seeing the doctor)
 Not enough information
 Poor quality information (not clearly written or spoken, not what the patient
wants)
 Adequate information communicated poorly (given too late or rushed or given
insensitively or when the patient was alone)
 Inconsistent or contradictory information from different sources.
49
APPROACHES TO THE CONSULTATION
The various general approaches to the consultation are - as here - often loosely
referred to as models, but in fact they are of very different status. Remember that
some are the result of intuition, some of empirical study - though the empirical
studies are themselves of mixed value. All, at least, are attempts to capture some
element which either does or should take place in the consultation.
Note in particular that these are not really competing models aimed at describing
the same phenomena: rather, they are all complementary...... and imperfect.
THE BIOMEDICAL MODEL:
1.
2.
3.
4.
5.
Take an accurate and relevant history (OBSERVATION)
Perform an accurate and relevant examination (OBSERVATION)
Make a provisional diagnosis (HYPOTHESIS)
Order and interpret the results of appropriate investigations (HYPOTHESIS
TESTING)
Make a definitive diagnosis (DEDUCTION)
Notes:
This is the classical medical diagnostic process. “Hypothetico-deductive” thinking
of the kind described here is still central to the doctor’s task in a consultation. In
any single consultation, the doctor may form, test and discard a large number of
diagnostic hypotheses based on the information (cues) s/he received from the
patient.
This is, however, a very incomplete account of the consultation.
 It is reductionist: the patients are seen and treated in terms of signs, symptoms
and diagnoses, and labelled accordingly
 It is doctor centred: there is no mention of the patient’s feelings, beliefs and
opinions, of any sharing of information or agreeing of a management plan
 It flounders when no objective physical disorder is unearthed
 It does not recognise the importance of non-verbal communication
 It omits the therapeutic use of the doctor-patient relationship
 It fails to recognise that a consultation can be one of a series - as is often the
case in general practice
 It over-emphasises the importance of decisions based on personal clinical
experience: these are apt to bias because of the limited number of patients that
any one doctor can experience
It is in response to these criticisms that other possibilities have developed, all
broadly holistic and all perceived as more modern in approach.
50
Here are a couple of the more important ones.
Pendleton et al
From observation, seven tasks were detailed which together form comprehensive
aims for the consultation:
1.
2.
3.
4.
5.
6.
7.
To define the reason for the patient’s attendance, including (I) the nature
and history of the problems (ii) their aetiology (iii) the patient’s ideas
concerns and expectations (iv) the effects of the problems
To consider other problems: (I) continuing problems (ii) at-risk factors
With the patient, to choose an appropriate action for each problem
To achieve a shared understanding of the problems with the patient
To involve the patient in the management and encourage him/her to accept
appropriate responsibility
To use time and resources appropriately (I) in the consultation (ii) in the long
term
To establish or maintain a relationship with the patient which helps to
achieve the other tasks.
Health Belief Model
The decision to consult depends on:
1.
The individual’s general interest in health matters, which may correlate with
personality, social class, ethnic group etc
2.
How vulnerable or threatened a patient feels him/herself to be to a particular
disease
3.
The individual’s estimation of the benefits of treatment weighed against
cost, risks and inconvenience
4.
Trigger factors such as alarming symptoms, advice from family or friends,
messages from the mass media, disruption of work or play.
This will be covered in more detail in the Health Promotion Session.
Additional Reading:
Textbook of General Practice, Stephenson. Pages 17-29.
“Clinical Method”, Fraser, 3rd Edition. Pages. 25-108.
“Notes for the MRCGP”, Palmer, 3rd Edition. Pages 68 – 74.
51
Weeks 2 & 6
Fri 10.45am – 12.45pm Dept. of General Practice
AUDIT
AIMS
To acquire the knowledge and skills necessary to successfully perform audit.
LEARNING OUTCOMES
 To define audit and understand its purpose.
 To outline the stages of the audit cycle.
 To describe the importance of topic selection and setting clear objectives.
 To design a simple audit.
 To demonstrate an appreciation of the potential benefits and difficulties of
performing an audit.
 To interpret data relating to audit.
 To differentiate between audit and research.
 To recognise the importance of teamwork and ethical principles in relation to
audit.
TEACHING METHODS
Small group teaching.
Practical exercises involving examples of audit.
Additional Reading
Pages 27 - 29, “Notes for the MRCGP”, 3rd Edition. Palmer
Pages 194-206, “Textbook of General Practice”, Stephenson.
Pages 1 – 14, “Medical Audit and General Practice”, Marinker
Pages 9 – 59 Principles for Best Practice in Clinical Audit [NICE] www.nice.org.uk
There will be additional written material provided at the beginning of the session.
52
AUDIT BASICS
Place of Audit - part of being a professional is the responsibility for
maintaining high standards of care. Audit is an integral part of that and is
expected of both Hospital Doctors and GP’s. Professionals are increasingly
accountable to the public... actions may have to be justified. Professionals are
aware of the need to maintain high standards of care; audit is a tool used to
improve clinical care. It should function to encourage genuine efforts to
understand what has happened in practice. It should promote learning and
shape change in the light of the reflections made.
Definition
Clinical audit is the attempt to improve the quality of clinical care by measuring
the performances of those providing that care, by considering the performance
in relation to desired standards and by improving this performance
Purpose of audit
It is to improve the quality of patient care
Audit is ...
 confidential
 educational
 systematic
 relevant
Audit involves ...
 change
 time
 working together
Audit needs ...
 enthusiasm
 structure
 a commitment to change if indicated
 integrity (honesty)
 consent of those participating
Research is about determining or verifying what constitutes best
practice.
Audit determines how well that best practice is being
implemented.
Clinical Effectiveness involves applying the best available knowledge (derived
from research, clinical expertise and patient preferences) to achieve the best
possible outcomes of care for patients. Audit is an essential part of the process
to ensure that patient care is being delivered as intended.
Clinical Governance is the framework through which NHS organisations are
accountable for safeguarding high standards of care and improving the quality of
their services (it is a system of corporate accountability).
53
INTRODUCTION TO AUDIT DESIGN
Topic
must be useful and feasible
Objective
what do you hope to achieve
keep it short and simple (KISS)
Criteria
markers of good practice that are measurable
Standards
what should be happening?
expected % of cases where criteria should be met
level chosen may be arbitrary
- for ideal care
- for reasonable care
- for minimum acceptable care
Often 100% standard is chosen, and then exclusions are
made.
identification of “cases” –who belongs in the group?
selection of “cases” without bias
appropriate size of sample / is sampling necessary?
Data availability and collection - What is happening?- decide what
information is needed to measure criteria
Pilot - Ensure it will work
Method
Sample
Analysis of data – Collate information into meaningful groups, comparison of data
with original criteria and standards set
Discussion - How do we get there? i.e. meet the standards set?
Outcomes – What have we achieved? What changes are to be implemented as a
result of the audit?
Review - consider again – Have we improved?
What Should
be happening?
Change
Introduced
What is really happening
54
SAMPLING
Why Sample?
To get an overview of the group of interest
To save time
To reduce the cost
Collecting data about every patient in a group of interest may not reveal any more
information about the group than a well-designed sample.
You have to decide if you want to review every patient individually (i.e. do a
census) or if you wish to study a sample of the group.
When might sampling not be helpful?
If the information you want / need comes from a computer search, and is readily
accessible, there is no real value in sampling – it takes no longer to do the whole
group.
A sample must be
Representative so that valid conclusions can be drawn i.e. without bias
Large enough to give realistic measure of performances.
It is important that the register / group of patients is as complete as possible.
Studies would show that patients not included are often not as well cared for as
those on registers.
Dealing with results after sampling
If you sample, you must apply your changes / improvements to the whole group;
i.e. not just ‘fix’ the sample.
55
AUDIT FORM
Rationale - why the audit is being done
Topic
Objectives
Target
Standard %
Criteria
Method - data - what is to be collected, is it available, in what form?
- identify patients - how/how many patients will you sample?
- who will carry out audit?
- pilot - will it work?
Data collection
Analysis
Outcomes / Action
Re-audit
What Should
be happening?
Change
Introduced
What is really happening
56
Week 5
Wednesday, 9.15am Dept. of General Practice
PRESCRIBING IN GENERAL PRACTICE
INTRODUCTION
Prescribing of drugs is one of the few legal rights conferred by registration as a
medical practitioner and with the considerable expansion in the pharmaceutical
industry in recent decades has become a more important mode of treatment than
ever before.
Prescribing by general practitioners in Northern Ireland accounted for expenditure
of approximately £366 million in 2005/6. This expenditure is increasing at a rate
several times greater than the general inflation rate.
Every year sees the introduction of 30 to 40 new drugs. Evidence of the
effectiveness of old drugs is constantly changing. Newly qualified doctors will,
during their professional career, see a complete change in the nature of
therapeutics and pharmacology. Doctors must learn to keep up to date with the
changing evidence and learn rapid and effective ways of updating themselves.
The volume of prescribing is also increasing. Each GP issues approximately 70
prescriptions per day, most of which are repeat prescriptions. The task of
recording, monitoring and issuing these prescriptions imposes a considerable
administrative burden on practices.
AIMS
The session aims to introduce the student to the principles of good prescribing, the
process of writing prescriptions, the availability of common references sources and
the treatment of common conditions in general practice.
OUTCOMES
The student should be able to
1.
2.
3.
4.
5.
Write a NHS prescription for a condition commonly encountered in general
practice.
Locate information to assist in the prescribing decision.
State the particular requirements for prescribing different legal categories of
drug.
Be aware of the potential influences on prescribing.
Be aware of the common causes of medication errors and adverse drug
reactions.
Recommended Reading
“Guidance on Prescribing” - Pages 1 to 25, British National Formulary.
Additional Reading
Pages 137 - 160, Textbook of General Practice, Stephenson.
Pages 75 - 85, “Notes for the MRCGP”, Palmer. 3rd Edition.
57
58
59
60
61
62
63
64
65
66
67
68
COMMON SYMPTOMS & “RED FLAGS”
Week 5
Mon 2.00pm Dept. of General Practice
AIM
To introduce students to a range of common symptoms seen in General Practice.
OUTCOMES
1.
Describe the causes of a range of common symptoms.
2.
Differentiate between common and less common medical causes of these
symptoms.
3.
Prioritise “Red Flag” associated symptoms or clinical findings which would alert
the student to a more serious problem.
TOPICS OF STUDY
 Acute Abdominal Pain in Adults
 Acute Confusion
 Acute Shortness of Breath
 Blood in Urine
 Chest Pain
 Headache
 Rectal Bleeding
 Sore Throat
 Swollen Glands
 Tiredness
Handouts will be given out in class during this session.
Reference: Symptom Sorter (1999)
Authors:
Keith Hopcroft and Vincent Forte
Publishers: Radcliffe Medical Press
69
MEDICAL RECORDS
High quality medical records are important in providing a good standard of
healthcare for patients. In the past records were often kept by each professional
group with little or no sharing of information. Medical records are now increasingly
multiprofessional and this is of benefit to the patient. Guidance written for doctors
by the GMC in “Good Medical Practice states” that ‘medical records should be
clear, accurate, legible and contemporaneous’. Patient records should report the
relevant clinical findings, the decisions made, information given to patients and any
drugs or other treatments prescribed.
This guidance also states that medical records must keep colleagues well informed
when sharing the care of a patient. When faced with a serious complaint or
litigation, many doctors wish that they had made more complete records.
Complaints can be difficult to deal with when records are incomplete. Similarly the
successful defence of a claim may be seriously hampered by records that are
illegible, inaccurate or incomplete (or indeed missing altogether). It is essential
therefore that doctors keep medical records to a high standard with complete
information both for the benefit of patient care and also to protect themselves.
What should good medical records contain?
Good medical records should allow another medical practitioner to make a clear
reconstruction of a consultation. They should be recorded in a logical way, whether
they are written or computerised records.
Records should include the following:

HISTORY – a summary of why the patient has presented including details of
symptoms, time-scales, associated illnesses, allergies and relevant social
history.

EXAMINATION – record any important findings, both positive and negative.
Always record details of measurements (eg. blood pressure “140/80”, not blood
pressure “normal”)

DIAGNOSIS –not only should a clear record of a diagnosis be made if possible
but also a record of how the diagnosis was reached. If differential diagnosis are
considered, record these and demonstrate why any were excluded.

INVESTIGATIONS – Record any that have been requested.
PATIENT INFORMATION
Record what information has been given to the patient including patient information
leaflets. Any medical treatment of patients has risks and benefits and they should
be not only discussed with the patient but also recorded.
70
This is also relevant for the consent process which is covered elsewhere in the
Study Guide. It may also be important to record when a patient has declined
information, when questions are asked by a patient and the answers given. It is
important to record details about the treatment advised including medication,
dosage, duration and treatment and any advice or warning given about treatment.
It’s also essential to record the follow-up information. This might include future
appointments, follow-up investigations and any referrals made.
When out in practice it is useful to review written medical notes recorded by a
number of different doctors to see the different styles of keeping records. Problems
commonly seen in medical records include the following:



Poor handwriting and therefore illegible notes
Incomplete records (eg TRxpenv it is assumed the patient has tonsillitis and a
course of penv has been prescribed. This record would not be deemed
acceptable by the professional body)
Lost records – it is common in both hospital and general practice for written
records to go missing if there are no back up copies records are permanently
lost.
RISKS ASSOCIATED WITH ELECTRONIC RECORDS
Some risks are the same as written records eg. over-use of abbreviations or
incomplete records but computerisation has introduced a number of new risks.
There is some concern that computerised records may be less comprehensive and
contain more abbreviations. Research to date has not proven this one way or the
other. Secondly misuse of Read Codes - these are a clinical coding system used
extensively in computerised health records to provide structure and allow
complicated searches for specific codes. However it is important that a code for a
diagnosis is only coded if it is definite eg. a patient querying angina may be
included in a list of patients with definite angina when a search is made. This could
have implications for research audit and also clinical management of that particular
patient.
BENEFITS FOR ELECTRONIC RECORDS
They are safe if basic advice is followed, available when required, readily retrieved,
readily analysed for audit research and quality assurance. They are also
convenient, improve clinical outcomes, protect privacy, reinforce confidentiality and
can have decision support such as use of templates, to manage patients, allow
automatic reports and support email information and electronic data interchange.
They also enable record transfer between healthcare professionals and provide a
comprehensive audit trail. In Northern Ireland all practices are computerised to
some extent and most practices although not yet paperless are certainly paperlight.
However the natural progression from using written clinical records to using
electronic means that some records are kept in a written and electronic format.
When dual records are kept this is the time which carries the greatest risk.
71
It is important that the electronic system is developed as quickly as possible and
that less and less dependence is put on the written record. When information is
only kept on the written record and not the computer record and does not match
then clinical errors may occur.
EXPERIENCE DURING CLINICAL ATTACHMENT
It would be useful to observe what information your GP inputs to the patient
records after a consultation. You should also volunteer to do this. Please make
sure you print your name. If a medical student in practice inputs either the
electronic or written records it is important to get the general practitioner or health
professional supervising you to check the entry you have made. With practice you
will be able to quickly summarise the relevant parts of the consultation and record
this adequately. This is a key skill in becoming a safe and competent practitioner
and any skills you learn in general practice can be applied to any other clinical
environment you find yourself in.
REFERENCES
Clinical Risks and Management of Primary Care: Authors – K. Haines, M. Thomas
Radcliffe Publishing ISBN 1-85775-869-2.
72
Weeks 1 & 5 Friday, 11.15am – 12.30pm Dept. of General Practice
Common Clinical Conditions
Daktar Daktar!!
You are a GP in a busy morning surgery in inner city Belfast. Your list of patients is as
follows – what would you do???
1. Miss Take , 26, (Lives in upper Malone)
Good Morning Doctor. I’m awfully sorry to bother you but I seem to have had intercourse
with my boyfriend - he drives a Porsche don’t you know – last night and appear to have
forgotten any protection. I have a very high powered job in law (ie legal secretary) and
could not possibly get pregnant.
Good general health.
LMP 2 weeks ago
Present method of Contraception – nil (Porche driving boyfriend cannot afford condoms)
No Known Drug Allergies
Present Meds – nil.
2. Master Green 3, (Lives Falls Road) Brought by mum
Darktar, Yer gonna have ta do summit about ‘is ear. ‘e’s been up all night an I aven’t had a
wink. It’s bin going on fer 3 days – e’s roasting one minute then freezin’ the next and the
ear pain started last night.
Previously well.
Temp 38.3
TM pink. No pus
No cervical lyphademopathy
3. Mr Muscle 37 (Works in a gym)
Well Hello Doctor, Do I know you from somewhere? I have a problem with
my back, but I’m sure you’ll be able to fix it. I was taking my step class last
night and I jumped off the step at the end while doing the splits – I’m, ever so
fit – and when I landed my back was sore just above my gluteus maximus.
What could I take to make it better?
PMH – nil of note
On examination – No neurological signs, no focal bony pain
73
4. Miss Red, 18 (Daughter of the Local ENT Consultant who taught you at QUB)
Sorry to bother you doctor, my dad sent me – he wants a second opinion on my throat. It’s
been sore for 3 days now and not getting any better. I’m doing my ‘A’ Levels next week
and I really have to do well – I want to do Medicine.
No PMH
Throat Red, some exudate
No cervical lympadenopathy
Temp 38
5. Mr Orange, 16 (Lives Sandy Row)
Right Doc! – How’s it going? Thing is, right, me spats are getting worse and me girl sent
me down ta get summit. She wants me ta use this poncy face wash but I thought you might
be able ta help.
Mild acne, no scarring.
6. Miss Lead, 21 (Medical Student)
Hello Doctor, I’ve just finished my Obs and Gynae attachment and I’ve been thinking
about starting the pill.
PMH – Nil
Current Meds
Smokes 10/day
LMP 3/52 ago
No Intercourse yet.
7. You’re about to call your next patient when the receptionist telephones you to say
there’s an urgent call on the line and she says it can’t wait until the end of surgery – It turns
out it is Miss Fit, 59, teacher, who feels she has the “flu”. She feels she has a fever – here
temperature is 37.3 and has been coughing and sneezing all day whilst shopping in Tesco
and playing golf. She feels she needs and antibiotic as she has to play Bridge tonight and
doesn’t want to infect the other players.
8. Miss Chief, 3 brought by mum,18
Doctor, I left a sample of urine in, like you asked the last time (child unwell, vomiting,
smelly urine) and it came back “positive” – What does that mean, is it serious?
No allergies
MSSU –
Significant growth of Coliforms
Sensitive to
Trimethoprim
Augmentin
Nitrofurantonin
Amoxycillin
74
9. Master Scratch 5, just started primary School, brought by mum (new age hippy)
Hello Dr, It’s Jonny – He’s been sent home from school because its leaping – they all have
nits! Will I have to cut all his lovely long hair off?
PMH nil
10. Miss Guided, 23 (just back from Holidays in India), Urgent Appointment
Good morning Doctor, thanks for fitting me in. I came back from Deli yesterday but the
whole time I’ve been there I’ve had diarrhoea. I think I need something to stop it.
11. Adam Dyper, 6 week old baby.
Daktar, he’s developed this really bad redness around the nappy area, is there anything I
can use to help this clear up?
On examination-red inflamed skin around nappy area with sparing of skin folds.
12. Amy Plaque, 30 year old secretary.
My psoriasis has really flared up, I can’t cope with this.
On examination- scaley plaques on limbs especially around extensor surfaces. Few plaques
at hair line also.
13. Miss Itch, 5years old, scratching skin especially around flexures. Her big brother has
hay fever.
14. Mr McFry, 56yr old publican
Dakta, I tell you what, I was in with the nurse, she told me I got high blood pressure, and I
needed to loose weight, what’s she on about??
BP 166/94
BMI 31
High weekly alcohol intake
Smoker 40/day for 40 yrs.
15. Miss P Anic
Just as your afternoon clinic is about to start, you get a call from a distressed lady. She is
caring for her mother at home, who has terminal cancer. She feels her mother has difficulty
going to the toilet and is straining. She also feels those new pain Killers are doing her no
good.
A Handout with Answers will be given out at the end of this session.
75
Week 5
Monday 3.15pm Dept. of General Practice
COMPLAINTS HANDLING IN GENERAL PRACTICE
AIM
To introduce students to complaints in the General Practice Setting.
OUTCOMES
To define what is a complaint.
To recognise good customer care and why it is important for any organisation.
To list the key recommendations of the Wilson Report.
To describe the basic complaint procedure in general practice.
To recall common reasons for complaint.
To list examples of good practice which may prevent complaints.
Definition of a complaint
Any expression of dissatisfaction which needs a response.
Task Force Good Practice Guide 1995
Business takes Customer Service very seriously. Satisfied customers cause less
stress and also take up less time. Finding a new customer costs 4-5 times more
than keeping the existing one: good customer care keeps contracts and also helps
to win new contracts.
Only 10% of customers, who have had unsatisfactory service, will complain.
Until fairly recently complaints were given little thought in the Health Service. To
many health care professionals they were and still are regarded as an nuisance.
There were many bureaucratic barriers to making a complaint. Patients had no
idea how to go about it and when a complaint got into the system the process was
rigid, slow and laborious.
This caused much unnecessary stress on both sides. Several years ago a number
of GPs who had complaints against them waiting in the system committed suicide
and there were calls from all sides for a radical overhaul.
The Wilson Report “Being Heard” 1992, recommended changes to handling
76
complaints in the NHS. A key recommendation was that Practices should establish
a clear resolution process at local level. This should allow the majority of
complaints to be dealt with quickly and to the satisfaction of both parties rather
than a long drawn out process.
Under their terms of service general practitioners are obliged to establish an inhouse complaints procedure for their practices. A Complaints Officer who may be
a doctor or other member of the staff administers it. Details of the practice based
procedures should be displayed in the waiting area and the practice leaflet.
A complaint should be acknowledged within 2 working days and generally an
explanation given within 10 working days.
If the complainant is not happy they can ask for an independent review which if
considered appropriate is looked at by an independent panel convened by the local
Health Board. If still dissatisfied the complainant can appeal to the Ombudsman.
The key features are that it should be easy to make a complaint and people should
know how to go about it. The process should be responsive, confidential, simple
and impartial. It should also be seen as being cost effective and above all, giving
the opportunity to improve the quality of care provided by the practice.
This certainly gives the patient important rights. However, rights have
responsibilities. For example, a patient has the right to make an appointment to
see a doctor but they have the responsibility to keep that appointment and see the
doctor.
An effective complaints system should put things right for individuals who have
received a poor service. It allows practices to learn from complaints and also to
make people aware that the practice cares how their service is perceived by their
patients.
Why do people complain?
They may simply want to be acknowledged. It can provide an outlet for emotions
and feelings. Often an apology or explanation is sufficient. Less commonly
redress and or compensation is sought and occasionally punishment is demanded.
Common reasons for complaints in General Practice
Failure to visit
Failure to diagnosis
Error in prescription
Failure to arrange
emergency admission
Delay in diagnosis
Failure to examine
25%
20%
8%
6%
5%
5%
77
Delay in visiting
Unsatisfactory attention
Failure to refer
Poor administration
Other
5%
5%
5%
5%
1%
Females are more likely to complain than males and complainants are usually less
than 45 years old. 50% of complaints are from individuals complaining on behalf of
someone else.
Saying sorry can prevent complaints in the first place. Indeed, most instances of
patient dissatisfaction never develop into a complaint because the doctor gives an
instant explanation and a courteous “I’m sorry”.
How to prevent a complaint with an apology
 Take all grumbles seriously no matter how unimportant or benign they seem.
 Acknowledge the grumble. For example, if a doctor arriving for the start of a
surgery is late he should apologise - if there was good reason he should say so
clearly.
 Own the grumble when it is directed at you; for example, if you, the doctor, have
been perceived to be rude you (not the practice manager or the senior partner)
should apologise.
 Learn from the grumble and take action to stop it happening again. Dealing with
minor grumbles carefully at the outset will prevent the becoming major
complaints.
 Saying sorry is about grasping nettles - very uncomfortable ones sometimes and swallowing your pride when necessary.
 Some kind of expression of regret is appropriate after any adverse outcome,
and sometimes, an explicit apology as well. For example, if you accidentally give
a child a vaccine containing the pertussis component when the parents have
specifically forbidden it, apologise immediately, there and then. Patients have a
right to know about such “hidden” accidents and letting them know is not
covering your back. A full explanation is part of good clinical practice and
should not be done just because patients will get the information by looking at
the notes.
 Patients are entitled to a prompt, sympathetic account and an apology when
appropriate - this is a recognition of their autonomy, their right to know about
themselves. Words such as fault, blame and liability should be avoided.
Additional Reading
Pages 253 - 254, “Textbook of General Practice”, Stephenson.
Pages 233 - 238, “Notes for the MRCGP”, 3rd Edition, Palmer.
78
Weeks 1 & 5 Fri 9.15am Dept. of General Practice
EMERGENCIES
This session introduces you to dealing with emergency situations away from the
relative security of the hospital setting.
Common emergencies will be covered
1.
Management of the acute myocardial infarction
2.
Status Asthmaticus
3.
Acute Febrile convulsion
4.
The unconscious patient and diabetic coma.
5.
Meningococcal meningitis
The second part of the session discusses paper work, equipment and drugs which
should be included in the Doctor’s Bag.
Recommended Reading
Handbook of Emergencies in General Practice 2nd Edition
79
ETHICS COURSE
There are two core sessions. These introduce and complement ethics teaching
timetabled formally in other courses within the Curriculum.
Week 5
Mon 9.15am Dept. of General Practice
Introduction to Ethical Theories & Principles and Ethical Dilemmas in General
Practice.
Medical Research, Ethical issues.
LEARNING OUTCOMES
To:
i)
ii)
iii)
iv)
v)
vi)
apply the moral issues arising in clinical practice and health services.
analyse clinical situations in terms of ethical principles and recognise the
ethical issues.
describe the main ethical theories and principles.
recognise the conflict between the various ethical principles and moral
issues within various clinical situations.
describe various moral arguments and understand the views of others.
an awareness of one’s own moral values and an understanding of their
basis.
Ethics is part of every day medical practice. Ethical issues will be discussed by the
Tutor as they present themselves in practice.
INTRODUCTION
Moral dilemmas are created by caring for people. The Medical Ethics course aims
to raise your awareness of these problems and try and arrive at working solutions
in a wide social context. Moral consciousness is not related to doctor status and
cannot be taught or passively acquired by reading alone. Its development requires
active participation in the debate.
DEFINITIONS
Ethics is fundamentally about “how best to conduct one’s life in the presence of
other lives”. Medical ethics has been defined as the obligations of a moral nature
which govern the practice of medicine. In one sense of medical ethics doctors
have been receiving medical ethics education at least since the time of
Hippocrates in the 5th Century BC. Thus the profession has accepted and
continues to accept a code of ethics – a set of moral principles to govern its
practice and it ensures that its members accept and implement this code.
80
From earliest times, various legal systems have incorporated some degree of
regulation of doctors. The Hippocratic Oath (4th Century BC) indicated the early
concern of the profession to regulate itself by laying down basic standards of
conduct, not only between the doctor and patient, but also between teacher ad
pupil. In the ensuing centuries the principles of Christian humanism dominated the
practice of medicine. Traditions of etiquette in public and private life gradually
evolved, coupled with those criteria of professional conduct which established the
physician’s position in society.
The changing attitudes of society and the major advances in medical science
during the 20th century has led to recognition of the need for an updating of these
simple ethical guidelines. This need was met initially by the Declaration of
Geneva, formulated by the World Medical association in 1947, supplemented by
Declarations on particular aspects of medical ethics, such as those concerning
therapeutic abortion, research, and torture and other human or degrading
treatment.
For some ethics means little more then etiquette; the accepted conventions of a
social role. Medical ethics in this sense means correct professional behaviour
which is passed on from older to younger practitioners by precept and example.
Sometimes when people talk about “medical ethics” they are talking about the legal
rights and duties of doctors and patients (we might call this “legal medical ethics”).
However, morality and the law are separate phenomena: actions which are legally
permissible may still be morally wrong and in extreme cases it may be morally right
to do something illegal.
As stated above, the term ethics is used as an alternative to morals or morality.
Thus we speak of an “ethical code” or a “moral code” and of “unethical” or
“immoral” behaviour. Medical ethics is not must ethics for doctors – it is ethics for
all those involved in giving and receiving health car. Here ethics refers not just to
socially acceptable behaviour, but to what is claimed to be morally good or bad,
morally right or wrong. This concept of medical ethics has been called “traditional
medical ethics” and refers to ordinary professional morality. In this introductory
handout, it is this concept of medical or health care ethics that is being considered.
As doctors we aim to discover, analyse and relate to each other the fundamental
concepts and principles of practical morality with a view to improving our practice
of health care. Medical ethics is part of moral philosophy, the academic discipline
concerned with the critical study of morality; seeking the fundamental principles,
norms or values which lie behind particular moral judgements.
SOME ETHICAL CODES
(i)
Hippocratic Oath
The Hippocratic oath was probably written in the 5th Century BC. A doctor
who takes the Oath swears above all to try to benefit his patient and not
harm him or her. He also swears never to divulge what he sees or hears in
the course of his profession. It is a patient-centred ethic hallowed by
tradition.
81
(ii)
The Declaration of Geneva.
This is the modern restatement of the oath drawn up in 1947 by the World
Medical Association and amended in 1973 and 1983. In it the doctor
swears to maintain the utmost respect of life.
Other codes of ethics followed:
(iii)
(iv)
(v)
(vi)
Sydney in 1968 defined the criteria of brain death
Oslo in 1970 discussed criteria for therapeutic abortion
Tokyo in1975 adopted guidelines for doctors concerned with torture and
punishment
Lisbon in 1981 discussed patient rights and confidentiality.
All these codes provide guidelines on specific issues, but they do not resolve
adequately the conflict between the claims of the individual and the requirements
of society.
TWO MAJOR TYPES OF ETHICAL THEORY
(i)
Deontological theories based on “rights and duties” of persons (deon is
the Greek word for duty).
In this group the consequences of actions are not taken into account. The
great religions expect obedience to moral rules that make no reference to
consequences (e.g. the Ten Commandments). It is impossible in a few
paragraphs to offer any sort of detailed explanation of the philosophical
thought behind these theories. Suffice it to say in practice that the orthodox
Christian view is that all human beings are morally equivalent and have
natural rights, a right to life, a right not to be killed and a moral duty not to kill
others. A common element in all deontological theories would seem to be
that rightness and wrongness, goodness and badness are treated as
intrinsic to actions of a particular kind (lying is bad in itself). Talk of rights,
duties and (perhaps) justice is more commonly met in (and perhaps more
suitable to) theories of this sort rather than those mentioned in the next
section.
(ii)
Theories of consequentialism
Consequentialist theories in contrast to those in the previous section judge
rightness and wrongness, goodness and badness by a factor extraneous to
the action itself, ie by reference to the consequences of such actions.
Actions are not right or wrong in themselves, but only in relation to the
consequences which follow from them. Such theories give no weight at all
to the motives of intentions of an agent when judging the morality of his
action. The best known of these theories is utilitarianism, the theory of
utility. This theory is about maximising happiness and minimising misery as
a consequence of action taken: that one’s actions are right insofar as
82
they tend to decrease happiness or increase misery ad morally neutral
insofar as they tend to do neither. As Raanan Gillon says “this idea,
encapsulated in the Benthamite slogan, “the greatest happiness of the
greatest number”, is the basis of all utilitarian theories of ethics”.
The most telling argument offered against utilitarianism is that sometimes
the interests of the individual may be over-ridden by the interests of society
as a whole as happens when the interests of the mother and society in
therapeutic abortion overwhelm those of the foetus.
THE MAJOR ETHICAL PRINCIPLES
(i)
(ii)
(iii)
(iv)
(v)
(vi)
(vii)
Beneficence – that one should do good to the patient.
Primum non nocere – the principle of non-maleficence that one should
do no harm. This principle is a cardinal principle already referred to in the
Hippocratic Oath.
Respecting the autonomy of the patient – this means that a patient
should be free to determine his own actions and give consent to the
treatment offered. Essentially autonomy is the capacity of the patient to
think, decide and act on the basis of such thought and decision, freely and
independently and without “let or hindrance”.
Truth telling – the principle of being honest is an important moral principle if
one is to preserve trust and confidence in the doctor-patient relationship. At
times, however, it comes into conflict with the principle of beneficence, e.g
deceiving a patient apparently for his own good.
Confidentiality - the principle of confidentiality between doctor and patient
is venerated in the Hippocratic tradition. It is not an absolute moral virtue,
but essential to maintaining rapport, trust and confidence in the consultation
and on-going doctor-patient relationship.
Preserving Life – Phillips and Dawson argue that maintaining respect for
life is not synonymous with preserving life at all costs. The principle of
maintaining respect for the sanctity of life at all times gives rise to many
modern moral dilemmas, e.g. the difference between killing and allowing a
patient to die, the ethics of therapeutic abortion and research on embryos.
Roman Catholic theology teaches that the human organism with a spiritual
life principle has a right to life. This right to life, while not superior to mother
or siblings, takes precedence over pain, distress, embarrassment or
economic damage to the life of the family. Such a moral principle opposes
the concept “pleasure is good; pain is bad” and conflicts with the principle of
utility, of which we shall hear much later.
Justice – principle of justice refers to the fair distribution of scarce
resources within society and may conflict with one’s duty to individual
patients.
83
OTHER MORAL DOCTRINES
(i)
Acts and omissions
This is the difference between acting and refusing to act (e.g. between
drowning someone by pushing them into deep water (action) and allowing
someone to drown, although one has the power to save him (refusal to act).
Some people maintain that the moral value of a piece of behaviour can vary
according to whether it is an action or an omission. They might say for
example that though it would be wrong to kill a patient by lethal injection
(action), it might be morally permissible to withhold food and allow him to die
(omission). Either way the consequences are the same; the man drowns,
the patient dies. So we should note that for a consequentialist the
difference between acts and omissions is morally irrelevant. Some
deontologists would claim that it is relevant.
(ii)
Ordinary and extraordinary means
This is a distinction between actions. In contrast to the previous one this
doctrine which would normally be accepted by consequentialists and denied
by (at least some) deontologists. Suppose for example that you have a
patient who is irreversibly comatose though not brain dead. A
consequentialist might argue that it was right (or at least permissible) to
keep the patient alive by ordinary means (e.g feeding) because such a
treatment does not cost very much and no one else is much harmed by it.
He might say on the other hand that it would be wrong to sue extraordinary
means (e.g a respirator) to keep the patient alive because such means are
expensive and if they are used to treat a hopeless patient, other patients or
potential patients who could benefit from them will be harmed by being
deprived of resources. A deontologist, however, who believes in the
sanctity of life might say that the means are irrelevant. We should try to
preserve life whatever the means and whatever the costs.
(iii)
Double Effect
“Double effect” refers to the situation where an action has two
consequences, one good and one bad, both known to the agent. For
example, giving painkillers to a patient will (a) relieve his pain (good), (b)
hasten his death (bad). Believers in “double effect” say that whether it is
right or wrong to give the painkillers depends on the intention of the doctor.
If the drugs are given in order to relieve the pain the action is right (or at
least permissible). If they are given with the purpose of hastening death the
action is wrong even though the outcome is exactly the same. If you have
understood the difference between consequentialist and deontological
theories it will be clear that the “double effect” doctrine will be unacceptable
to a consequentialist, since for him intentions are morally irreverent.
Deontologists, however, who believe that intentions are important, will
discern a moral difference.
84
Process
Interactive talk based on ethical dilemmas in general practice. Small group work
on ethical problems with feedback to plenary session.
Assessment
Some of the communication stations in the OSCE will address ethical issues.
Additional Reading
Talking to patients about cancer. BMJ 21st Sept 1996 p. 669-700.
Consent BMJ 1985: 291; 1700-1.
Good Medical Practice, GMC.
Confidentiality: Protecting and Providing Information, GMC.
Seeking Patients’ Consent: The Ethical Considerations, GMC.
Maintaining Good Medical Practice, GMC.
Serious Communicable Diseases, GMC.
85
Weeks 1 Tues, 9.15am, Dept. of General Practice
Weeks 5 Tues, 9.15am, Dept. of General Practice
EVIDENCE BASED PRACTICE
AIMS
To enable the student to (1) develop their understanding of scientific method in
medical research and (2) evaluate written reports of research evidence relating to
the practice of medicine.
OUTCOMES
The student should be able to:
(1)
identify the objectives of a reported research activity
(2)
list different types of study design in medical research
(3)
identify study designs appropriate for use in studying different clinical
problems
(4)
list factors which may influence the interpretation of research findings
(5)
apply research evidence to everyday medical practice.
CONTENT OF COURSE
You will be given a short talk to introduce this part of the curriculum. Critical
appraisal is one of the aspects of general competence which it is recommended
that the undergraduate medical student should develop. One aim of the new
undergraduate curriculum is to create a critical and enquiring doctor, willing and
able to continue learning throughout his/her professional career. By appraising
their own work and that of others, as reported in medical literature, doctors should
seek to continually improve the quality of their medical care.
In small groups you will discuss with a tutor the definitions and meanings of
words commonly used in medical research. Guidelines will be given for an
appropriate method of assessing written information with regard to evaluating its
relevance to clinical practice.
There will then be an opportunity to put these guidelines into practice by
assessing a written report both individually and as a member of your small group.
Within the group you should discuss your individual assessments and agree a
conclusion about the value of the evidence presented. Tutors will facilitate your
discussion and help you to interpret the research findings.
There will be one further formal opportunity to practise your skill in critical
appraisal later in the course but you should remember also to relate your learning
in this part of the course to what is presented to you in the part of your course
concerning audit.
ASSESSMENT
Assessment takes place as part of the Final MB examination. You will be given
published research articles and asked to critically appraise them. Scoring of your
answers will be based on the objectives of the course. Currently this assessment
consists of a two-hour written paper.
Additional Reading
Pocket Guide to Critical Appraisal, Crombie
BMJ Publishing Group.
86
Weeks 5 Tues, 2.00pm Dept. of General Practice
HEALTH PROMOTION
This session will examine an effective interviewing technique (incorporating a
model of behaviour change) on how to facilitate behaviour change and to discuss
the possibility of incorporating the technique into daily practice.
Effective approaches to helping people change an unhealthy behaviour will be
discussed and in particular the technique of Motivational Interviewing used in
conjunction with Prochaska and DiClemente’s Model of Behaviour change.
There will also be an opportunity for you to practise the technique in small groups
using various case scenarios.
On completion of the session perhaps your beliefs and attitudes to Health
Promotion will have acquired a new perspective. Doctors have an important role in
delivering health care. This session may help you become more effective when you
take on the role.
A resource pack will be given to you at the beginning of the session.
AIMS
The aim of this session is to enable students to explore and to practise the
technique of Motivational Interviewing in conjunction with Prochaska &
DiClemente’s Cycle of Change, as a tool to facilitate behaviour change.
OUTCOMES
At the end of the session students will have
1. to describe Prochaska & DiClemente’s model of behaviour change;
2. memorize the principles and strategies of Motivational Interviewing;
3. demonstrate the technique of Motivational Interviewing to help people change
an unhealthy behaviour;
4. judge how this technique of Motivational Interviewing could be integrated into
practice.
87
THE WORLD HEALTH ORGANISATION
The World Health Organisation (WHO) established in 1948, is a specialist agency
of the United Nations with primary responsibility for international health matters and
public policy. A total of 165 member states provide a budget for the work of WHO.
WHO has no legislative authority but it uses its considerable status to influence
health policy worldwide.
HEALTH PROMOTION IN CONTEXT
Health Promotion first appeared as a term and concept in 1974 when the Canadian
Minister of National Health and Welfare, Marc Lalonde, published “A New
Perspective on the Health of Canadians” (Lalonde 1974). It introduced into public
policy the idea that all causes of death and disease could be attributed to four
discrete and distinct elements:




inadequacies in current health care provision;
lifestyle or behavioural factors;
environmental factors;
bio-physical characteristics.
It was felt that improvements within the environment and in behaviour could lead to
a significant reduction in morbidity and premature death. As a result of this report,
emphasis in public policy shifted from treatment to the prevention of illness and the
promotion of health.
The Ottawa Charter (See handout)
The first international conference on health promotion was held in Ottawa, Canada,
in November 1986. Dialogue between lay people, health and other professional
workers,
representatives
of
governments,
statutory
and
non-statutory
organisations; politicians and administrators resulted in agreement on the Ottawa
Charter for Health Promotion.
Five major challenges for health promotion were identified:





to build healthy and integrated public policies;
to create supportive environments;
to strengthen community participation and action;
to develop personal skills;
to reorient health services.
88
These five action areas provide a framework for the delivery of health promotion
programmes. The Ottawa Charter also included three process methodologies
through which people could begin to take control over and improve health. These
were:
 advocacy
 enablement
 mediation
OTTAWA CHARTER
1. Build healthy public policy
To make the healthier choice the easier choice for individuals as well as policy
makers.
2. Create supportive environments
To enable us to take care of each other, our communities and our natural
environment.
3. Strengthen community action
To enable communities to set priorities and make decisions for better health.
4. Develop personal skills
To enable people to exercise more control over their own health and
environments and make choices conducive to health.
5. Reorient health services
To lead to a change of attitude and organisation of health services in order to
respond to the needs of the population.
BEHAVIOUR CHANGE
THE STAGES OF CHANGE MODEL
Prochaska almost 20 years ago began researching how – and why – people
change, he was confronted by a bewildering array of therapeutic systems, each
claiming to provide answers, but no one significantly more effective than the other.
Over the years Prochaska and his team have studied thousands of self-changers
and developed the Stage of Change model which has now been accepted as a
good model to conceptualise the way that people alter their behaviour.
The stage of change model has been used mainly as a basis for primary care
training for health education. The model allows for relapsing behaviour and redirection of programmes. It has potential but does require training in assessment
89
of clients’ stage of behaviour change. It is basically a one - one intervention,
building on counselling interventions to facilitate individual change. The influences
of the wider environment on the individual must be integrated into each stage of
the model to ensure that interventions are relevant to the client’s life experience.
Stages of Change
Prochaska and DiClemente have analysed motivation to change across a wide
range of problem areas and have identified five major stages of change.
(Prochaska and DiClemente 1984)
The system has five stages, each one of which has to be gone through for change
to be long lasting.
The first is called Precontemplation. In this stage, a person has no real desire to
alter their behaviour. In fact, they resist change. They don’t want to think, talk, or
read about their problem because they feel the situation is hopeless. Prochaska
states that they are often demoralised and in denial. For example, a young drinker
may consume five pints every night but does not believe that they have a drink.
In the second stage, Contemplation, people acknowledge they have a problem
and think about how to solve it. The same drinker might perceive a link between
marital problems or other pressures and alcohol consumption.
When perception of the costs and benefits of behaviours begins to alter, a person
may move into the Preparation stage. This involves setting a definite date for
change and planning how to do it. Only then are they ready for the Action stage.
Last, but not least, comes Maintenance, when people work at sticking to their
goals and attention is turned towards prevention of relapse to less healthy
behaviours.
The key to the programme is first deciding what stage the person is at, and then
using whatever process of change best applies to that stage. The majority of
people who are likely to be targets of health promotion initiatives are in either the
Precontemplation or the Contemplation stages.
Health promotion interventions should be designed to be appropriate to each
individual’s stage of change. For example, in Precontemplation, consciousness
raising is vital to the person becoming aware that they have a problem. However,
in Action, interventions such as developing coping skills, countering techniques –
substituting healthy responses for problem behaviours – and rewards that reinforce
good behaviour are relevant and useful.
Most people intuitively know what works best for them.
90
This model of change has been shown to work. The cycle allows for change to
occur but with recognition of the very real possibility of relapse to the former
behaviour. For example, most people attempting to stop smoking will relapse. They
may become demoralised and may not want to consider a further attempt.
Fortunately, the vast majority do eventually progress again to the action stage.
ROLE PLAY FOR SMALL GROUP WORK
Three persons in a group and swap roles every 8-10 minutes.
1. Patient
2. Doctor
3. Observer
1. Patient
Contemplator
Smoker / excessive alcohol consumption / weight reduction / Drugs
Choose situation that you personally can associate with to make it easier to role
play.
Suggestions
 Excessive alcohol: Business person, busy working and social life, Caught
speeding, alcohol level just under the limit.
 Smoker: Attends for inhaler for wheezy chest. Brother's inhaler worked well for
him/her. Father died of Lung Cancer 10 years previously.
 Overweight: BMI >32 Routine BP check, elevated at 175/110. Strong Family
History of Ischaemic Heart Disease.
2. Doctor
Use yellow sheet as prompt
No more than 2 sentences at a time
Once you get comfortable with technique, try advice giving
3. Observer
Interrupt process each time you feel statements are inappropriate or the doctor is
‘advice giving'. Feedback to doctor/patient as interview proceeds
91
PRACTICAL STRATEGIES IN MOTIVATONAL INTERVIEWING
A.
The Good Things and The Not-So-Good Things
Aim
To explore people’s feelings about the behaviour in question, without imposing on
them any assumptions about it being ‘a problem’. They, rather than you, identify
problem areas or reasons for concern and change.
Function
Often useful soon after first raising the subject, this strategy can serve the following
functions:
1. to explore the behaviour in a non-threatening manner
2. to build rapport, and understand the context of the behaviour
3. to minimise resistance because you talk about the positive thing first, then
discuss “less good things” rather than ‘problems’ or ‘concerns’, which allows the
person to identify problem areas without feeling labelled
How to do it
1. Ask open-ended question about the positives:
“What are some of the good things about your use of …….. ?”
“What do you like about ……..?”
These usually emerge rather quickly. Use reflective listening and summarise as
necessary.
2. Ask open-ended question about the negatives:
“What are some of the less good things about your use of ……..?”
“What about the other side?”
“What do you not like about ……..?”
Elicit these one by one, finding out why the person thinks these are ‘less good
things’. Open-ended follow-up questions are useful here such as:
“In what way does this affect you?”
“What is it that you don’t like about this?”
“Can you give me some examples?”
3. Offer a summary reflection as succinctly as possible, drawing together the good
things and the less good things. Use ‘you’ language, and give the person time
to respond. For example:
“So, using alcohol helps you relax ….. you enjoy drinking with friends, and it
seems to help when you’re feeling fed up. On the other hand, you
sometimes feel controlled by the stuff, you wonder if you’re harming your
health, and on Monday mornings you find it hard to do anything at work.”
92
Reminders
Avoid using words like ‘problem’ or ‘concern’ unless the person does.
Don’t assume that a ‘less good thing’ is a concern for the person.
Keep the task at hand, and avoid raising new topics or ideas of your own.
When the person offers concerns, consider using the ‘Exploring Concerns
Strategy’.
PRACTICAL STRATEGIES IN MOTIVATONAL INTERVIEWING
B.
A Typical Day
Aim
A good general strategy to explore the context in which the behaviour occurs, by asking
the person to describe a typical day or session in detail.
Function
This can be useful soon after raising the subject, particularly with people who seem less
concerned. The interviewer raises no problems, concerns, or hypotheses. This strategy
can build rapport, and aids in assessment including evaluation of readiness to change.
How to do it
1. Explain the purpose and define a time-frame
“Can we spend the next 10 minutes talking about your …… so that I can
understand better how this fits into your everyday life?”
2. Locate the day / session to be described
“Think of a fairly typical recent day (time) which would give a good picture of
how you use …… Can you think of one?”
3. Ask for a detailed description
“I’d like you to take me through this day, a step at a time, and tell me how
…… fits into -your day. You woke up at …”
4. Follow the person’s description using reflective listening and simple open
questions
“What happened then?”
“How did that make you feel”
93
Reminders
Avoid raising ‘concerns / issues’ of interest to you but not to the person.
Watch the pace. Push gently forward if going too slow, but don’t rush, and slow the
person down if going too fast.
If the person raises a problem, acknowledge it but continue on if reasonable,
agreeing to come back to it later.
PRACTICAL STRATEGIES IN MOTIVATONAL INTERVIEWING
C.
Exploring Concerns
Aim
To help people express for themselves what concerns they have.
Function
This is an important strategy, often the foundation for building motivation for
change. It highlights elements of ambivalence, and can lead to the generation of
discrepancy… a sense of discomfort … which can often precede a decision to
make change. This strategy can only be used with people already expressing
some concern, or who have opened up an area of concern (for example, in
response to the “Good things and Not-so-Good things”)
How to do it
1. Explore concerns on at a time. To open up discussion, ask an open question
like:
“What concerns do you have about ……..?
“In what ways has …….. been a problem for you?”
It can also be useful to ask about how / why others have been concerned. Explore
the concern in detail. Use follow-up questions, ask for examples, and use reflective
listening. Summarise the concern in ‘you’ language.
2. Move on to the next concern with an open question:
“What other concerns do you have about ……..?”
“How else has …….. worried you?”
“In what other ways has …….. been a problem for you?”
“What concerns you most about this?”
Again, explore in detail and summarise, as above. It can be useful in such
summaries to incorporate ‘the good things’ as well, particularly if resistance is
encountered. “So on the one hand you like …….. because …….., and you also are
concerned that …..
94
3. When all concerns seem to have been elicited, summarise them in ‘you’
language that captures both sides – the good things and the concerns.
Reminders
Don’t rush. Use simple open questions and reflective listening. The person may
feel ‘inefficient’, but in fact it is the best way to make rapid headway.
Don’t wander too far from exploring concerns.
The discrepancy that is often highlighted is this strategy can lead to discomfort.
Tears are not uncommon. Be supportive, and don’t rush the person into a decision.
Let the person raise the topic of change.
95
Weeks 1 & 5 Thurs, 9.15am – 10.45am. Dept of General Practice
CHILD HEALTH IN GENERAL PRACTICE
AIM
(a)
To provide students with background into the scope of child health care
provision in family practice before taking part in the attachment.
(b)
To compare and contrast the roles of the general practitioner and the
paediatrician in the provision of child health care.
OUTCOMES
1.
To describe in what way preventative medicine can be applied to child care.
2.
List the primary vaccination schedule.
3.
Describe the components of a comprehensive developmental surveillance
programme for the under 5s.
4.
List the common diseases of childhood.
5.
To show the ways childhood diseases present in general practice.
6.
Compare how the approach to paediatric history taking and examination
differs from adult practice.
PROCESS
 Presentations
 Small group work
96
Child Health Issues
Child Health in General
Practice
 Preventative
Medicine
Surveillance
 Family Health
 Common diseases of Childhood
 Common problems in children
 Developmental
Dr. Kieran McGlade
Department of General Practice
Queen’s University, Belfast
Department of General Practice
Preventative Medicine in Child
Care
 Ante-natal
care
Childhood Immunisations
 Primary
ante-natal screening
 folate supplements and nutrition

 Neonatal
immunisations given at 2, 3 and 4
months
 Primary immunisations
care

environment
 feeding


Mumps and Rubella vaccine in
second year of life
Department of General Practice
Queen’s University, Belfast
Department of General Practice
Things not to miss
 Hypothyroidism
dislocation of the hip
 Retinoblastoma
Syndrome
Department of General Practice
Developmental Surveillance
Queen’s University, Belfast
assessment of a child’s
developmental progress with the aim of
detecting pre-symptomatic disability
 Screening procedures should be brief,
simple, cheap and reliable
 May be combined with immunisation visits
- if so, screening should be done first
Department of General Practice
What do we look for in
developmental screening?
 Normal
growth pattern
Height, weight, head circumference
 Achievement of developmental milestones
(indicating neurological development)
 Gross motor function
 Fine motor function and vision
 Hearing and speech
 Social behaviour and play

Department of General Practice
Queen’s University, Belfast
 Periodic
 Phenylketonuria
 Down’s
Diphtheria, Tetanus, Pertussis and HIB
Polio drops
 Measles
 Immunisations
 Congenital
Queen’s University, Belfast
Queen’s University, Belfast
Queen’s University, Belfast
How do we carry out
developmental screening
 Often
in a clinic
and Health visitor
 Direct observation
 Listening to parents / carers
 GP

History is important - we want to find out what
the child can and cannot do
 Specific
tests
Department of General Practice
Queen’s University, Belfast
97
Developmental Screening
Tests 6 weeks
 Head
Control
usually achieved by 6 weeks
 Moro response
 present at 6 weeks, gone by 6 months
 Gaze fixes in mother’s face, follows a brightly
coloured object past the midline
 Rattle or bell, 15cm at ear level - quietens or turns
to sound.

Department of General Practice
Queen’s University, Belfast
Family Health
Family Life Cycle Stages
 The
new couple
of first child
 School age family
 Adolescent family
 Launching family
 Empty nest
 Ageing family
 Birth
Department of General Practice
Queen’s University, Belfast
Common diseases in
Childhood
Group Tasks
 Nappy
 List
the commonest diseases in childhood.
 List
the commonest presenting symptoms in
childhood.
Department of General Practice
Queen’s University, Belfast
rash
Respiratory Tract Infection
 Infectious Diseases
 Urinary tract infection
 Asthma
 Eczema and other rashes
 Trauma
 Upper
Department of General Practice
Infectious diseases and
common infections
Common Problems
 Measles
 URTI
 Sore
 Mumps
 Otitis
 Headaches
 Rubella
 Pertussis
 Chickenpox
Department of General Practice
media
 Tonsillitis
 UTI
 Gastro-enteritis
 Herpes stomatitis
 Meningitis
Queen’s University, Belfast
Queen’s University, Belfast
tummy
 Rashes
and fevers
wetting - enuresis
 Behavioural problems / school refusal
 Cough
 Diarrhoea and vomiting
 Bed
Department of General Practice
Queen’s University, Belfast
98
Group tasks (2)
how you might manage the
following scenarios:
On observing a consultation with
a child - questions to ask yourself
 Discuss
A six month old baby presenting with a fever
 A five year old presenting with a cough
 A nine year old with an itchy rash on her trunk.
 A ten year old child with recurrent headaches

Department of General Practice
Queen’s University, Belfast
Summary
 Child
health is an integral part of GP.
embraces prevention, diagnosis and treatment.
 A knowledge of the family is an important key to
the understanding of child health problems.
 Flexibility of approach, attention to detail,
observation and listening to children and parents
are of crucial importance.
 It
Department of General Practice
Queen’s University, Belfast
Reference:
A Textbook of General Practice, 2nd Edition.
Pages:
80 – 81
108 – 109
146 – 148
286
 Who
is the patient?
is the reason for the consultation?
 What opportunities does this consultation
present?
 Is the consultation structured any differently
from an adult consultation?
 What
Department of General Practice
Queen’s University, Belfast
99
EASTERN HEALTH AND SOCIAL SERVICES BOARD
CHILD HEALTH SYSTEM
SURVEILLANCE PROGRAMME
6-week Developmental Surveillance
Doctor/GP/CMO - Cephalo-caudal examination:
Weight - Length - Head Circumference
Fontanelle:
anterior
posterior
Eyes:
appearance
red reflex
fixing
following
smiling
Ears:
hints for parents
startle
pinnae
screening for children with “at risk” factors
Palate
CVS:
auscultation
femorals
Respiratory System
Abdomen:
liver, spleen
Hernia
Labia/testes
Hips:
(?) CDH
Hands
Feet
Spine
Skin
Developments:
tone
posture
head control
graso
moro
behaviour
100
EASTERN HEALTH AND SOCIAL SERVICES BOARD
CHILD HEALTH SYSTEM
SURVEILLANCE PROGRAMME
7-month Developmental Surveillance
HEALTH VISITOR:
Height
Weight
Physical:
general observation
Hips:
(?) CDH
Testes
Hearing:
distraction test
Vision:
observation and enquiry
(?) strabismus
Locomotor:
sits with support
rolls prone to supine
weight bearing
Manipulation:
reaches out for objects
transfers cube from one hand to another
Speech/Language: babbling
Social behaviour:
smiles and laughs
Health promotion:
nutrition and weaning
dental
play and social stimulation
safety
etc.
101
EASTERN HEALTH AND SOCIAL SERVICES BOARD
CHILD HEALTH SYSTEM
SURVEILLANCE PROGRAMME
18-month Developmental Surveillance
HEALTH VISITOR:
Vision
}
observation and parental enquiry
Hearing }
Physical examination:
general observation
Developmental screening undertaken by parental enquiry and observation and not
by formal testing.
Locomotor:
gait
walking unaided
Manipulation:
pincer grasp
Speech/Language: understands simple commands
Social Behaviour
Health promotion:
N.B.
discipline
safety
social stimulation and play
dental
etc.
Height and weight only if indicated
102
EASTERN HEALTH AND SOCIAL SERVICES BOARD
CHILD HEALTH SYSTEM
SURVEILLANCE PROGRAMME
27-month Developmental Surveillance
HEALTH VISITOR
Speech and language screening using proposed amended Cherryville test.
Health promotion.
Hearing screening of children “at risk”.
4-year Pre-School Developmental Surveillance
GP AND HEALTH VISITOR
Height
Weight
General physical
development
Testes
Hips
Co-ordination Gross motor and
Fine Motor:
observation
Language:
observation
Social:
toilet training
Screening for vision and hearing if parental/professional concern.
Health promotion:
Preparation for
School Health
103
MULTICULTURAL ISSUES IN PRIMARY CARE
Week 5
Mon 4.00pm Dept. of General Practice
INTRODUCTION
Northern Ireland is becoming increasingly multicultural with the arrival of migrant
workers, asylum seekers and refugees. General practitioners are facing the challenge
of developing new ways of providing care to communities with different health needs
and different understandings of health. While asylum seekers and refugees face some
health problems that are specific to them, they are often similar to other deprived or
excluded groups. The challenges of providing healthcare to a culturally diverse patient
population include having sufficient knowledge about Northern Ireland’s main cultural
groups while avoiding stereotypes as lack of such knowledge can lead not only to
personal distress and offence but may have serious consequences for treatment.
Good communication is also essential for an effective consultation. The use of
interpreters in General Practice in Northern Ireland varies. This may reflect a lack of
experience of use, an under-estimation of the need for interpreters, concerns over the
cost or about the extra time needed when using an interpreter (ICGP 2005). Providing
cultural competent health and social care is an imperative as patients expect
practitioners to provide care that is sensitive to their cultural beliefs and values,
appropriate to their needs and free from discrimination.
AIMS
This session aims to increase students’ sensitivity to the health and social needs of
patients from different cultures.
LEARNING OUTCOMES
At the end of this session students should be able to…..
 Describe the effects of different cultural and social practices for doctor-patient
interactions with particular reference to different naming systems, health
beliefs, religious beliefs and collectivist societies.
 Discuss barriers to communicating with patients from different cultures
 and the role of the interpreter.
 Describe the health needs of asylum seekers and refugees.
Handouts will be given out in class during this session.
Reading
Crowley P. (2003) General Practice Care in a Multicultural Society. www.icgp.ie
Cheal AC and Fine BP. (2005) Resource Pack to help General Practitioners and other
Primary Health Care Professionals in their work with Refugees and Asylum Seekers.
Lambeth Primary Care Trust website.
Irish College of General Practitioners. (2005) A Guide to Interpretation Services and
Cultural Competency. www.icgp.ie
Lloyd M and Bor R. (1996) Ch: 7 ‘Communicating with patients from different cultural
backgrounds’ in Lloyd M. and Bor R. Communication Skills for Medicine. London: Churchill
Livingstone.
104
Some Modified Essay Questions are listed. We suggest you
do the questions just in the time scale indicated and then
check your answers with the Marking Schedule listed.
105
You are a GP in an urban health centre with access to laboratory and Xray facilities
at the district general hospital one mile away. You have an active primary care
team working with you.
30 mins
PART 1
120
Mr Albert Torrens is a 45 year old man whose wife telephones you, his general
practitioner, one morning at 2.00 am. She tells you that he has severe pain in his
stomach and is vomiting.
Q.1
How do you respond?
You arrive at the home and find Mr Torrens looking shocked, pale and sweating.
He is sitting upright, leaning forwards.
Q.2
What further questions would you ask?
Q.3
What are your initial major differential diagnoses for a
patient with severe non-colicky epigastric pain? List 3.
106
Q.4
Describe your examination of Mr Torrens.
You admit Mr Torrens to the local hospital as a surgical emergency.
Q.5
What investigations do you think should be done when he
arrives there?
After investigations Mr Torrens is diagnosed as having acute pancreatitis.
Q.6
What are the common causes for this condition? List 4.
Mr Torrens settles with conservative management and is discharged home a week
later. His wife and 15 year old daughter, Alison, bring his discharge note to the
surgery. Mrs Torrens mentions that Alison has developed a cough and seems to
be short of breath with exertion.
Q.7
What salient features would you wish to consider in taking
a history and examining Alison?
107
You diagnose that she has a chest infection and decide to give her an antibiotic.
Q.8
What specific question will you ask before writing the
prescription?
Q.9
Write a prescription for an antibiotic for Alison.
Mr Torrens brings Alison back for review two weeks later. She is well again but he
complains of vague right sided pain and tells you that his urine has become very
dark.
Q.10
What do you specifically look for on clinical examination?
You check his liver function tests which confirm that he has acute hepatitis. He
admits to a recent heavy consumption of alcohol.
Q.11
What further questions would you ask him?
108
He returns to see you 4 weeks later with obvious weight loss and deep jaundice.
You arrange his admission to the local hospital's medical ward.
Q.12
What investigations would you expect to be done and what
abnormalities would support the diagnosis of alcoholic
hepatitis?
Q.13
How would you expect him to be treated?
A week later he is discharged and you visit him at home.
Q.14
What topics would you cover during the consultation?
Over the next 6 months you see Mr Torrens several times, he claims to feel well
and to continue to abstain from alcohol. Then, during morning surgery, you receive
a phone call from Mrs Torrens. She has found her husband lying on the floor and
cannot rouse him.
Q.15
What questions do you ask her on the phone?
109
You go and see him. He is obviously jaundiced and smells of alcohol. He is
responsive to painful stimuli.
Q.16
How do you manage the situation?
In hospital his level of consciousness rises but despite treatment he develops
fulminant liver failure.
Q.17
What complications do you expect to develop?
Ten days later Mr Torrens dies. You are informed of his death by hospital staff
who tell you Mrs Torrens is very distressed. You go and visit her in her home.
Q.18
How would you plan to manage the consultation?
110
PART 2
68
Mr Brown (aged 35) and his son, John (aged 4/12) arrive for their appointment.
The health visitor has diagnosed that John has atopic eczema affecting mostly his
face and flexors.
30 mins
Q.1
What advice would you give Mr Brown on
(a)
Prognosis
(b)
Non pharmacological advice. List 6 areas.
(c)
Steroids. List 6 areas.
111
Mr Brown asks for his blood pressure to be taken. Your nurse had taken it that
morning obtaining a reading of 180/100. You obtain a pressure of 150/95.
Q.2
Give reasons as to why the two readings differ. List 5.
Q.3
Describe specifically your examination of Mr Brown.
Q.4
What simple tests would you carry out?
Q.5
Discuss non-pharmacological management and advice with
Mr Brown.
112
Q.6
If Mr Brown's pressure did not settle suggest a pharmacological
plan of therapy.
Your nurse says that she would like an HIV test.
Q.7
What areas would you cover in your pre-test counselling. List 10.
113
PART 3
80
Marion Green, who is twenty, comes to see you almost in tears as she has
frequency and dysuria yet again. Since she became sexually active 18 months
previously the problem has become worse. She has had about eight episodes in
the last year. When the problem started MSU's were regularly done and coliform
infections identified on nearly all occasions. The symptoms do respond to
antibiotics but have recurred so frequently she now dreads intercourse and this is
producing difficulties with her present boy-friend.
Q.1
What areas would you wish to discuss with your patient?
Q.2
Apart from E.coli what other organisms are implicated in Urinary
Tract Infection? List 4.
Q.3
Which findings on dipstick analysis are a good indicator of
Urinary Tract Infection.
Q.4
How would you propose to manage the condition on this
occasion? Give the dosage of drugs you might use.
114
Q.5
What other prophylactic medicine might you also prescribe at this
time?
Marion returns in 10 days and tells you her symptoms have cleared up as is usual,
however, she is worried about recurrence.
Q.6
How would you plan to manage the patient at this stage?
Several years pass before you see your patient again. She had left your practice
but has now returned as a young wife and mother. Her husband is a local
business man who travels on business a lot. She asks you for a prescription for
"thrush" and tells you she has had several episodes in the last year and it usually
responds to Canesten.
Q.7
Is investigation essential at this stage? Give your reasons for
investigating or not investigating. If you do decide to investigate
what would you do?
Q.8
What organisms infecting the female genital tract can and cannot
be isolated with certainty in the setting of General Practice?
115
As expected Candida is found in this case.
Q.9
What treatment would you prescribe for this episode and what
strategies might be adopted to ease her recurrent attacks?
116
PART 4
66
Mrs Bailey, a 45 year old school teacher brings her son John to see you. He is
eight years old. The presenting complaint is one of a persistent dry cough which is
annoying the whole family.
30 mins
Q.1 What areas would you wish to cover in the preliminary history?
Q.2
What examination would you carry out?
You decide that John probably has asthma and give him a salbutamol inhaler to
try.
Q.3
What advice do you give to John and Mrs Bailey and what general aims
should you have in mind when treating children with asthma?
117
You ask John to come back for a review in one week's time, however, he does not
attend for another month or so. The salbutamol inhaler worked very well initially,
however, over the past week he has been getting more and more wheezy and out
of breath. Two days ago it was so bad he ended up in the A&E department of the
local hospital and was given nebulised salbutamol which rapidly alleviated his
symptoms.
Q.4
What might you include in your management now?
John does well on his regular inhaled steroid. However, several months later you
are called out in the middle of the night to his house. He had just returned from a
school camping trip that evening when his asthma became much worse. Mrs
Bailey is very anxious and says he is having his worst ever attack. You decide to
call round immediately.
Q.5
What are the features of a severe asthmatic attack in a child?
Q.6
What treatment would you consider for a severe asthmatic episode?
118
ANSWER & MARKING SCHEDULE
You are a GP in an urban health centre with access to laboratory and Xray facilities
at the district general hospital one mile away. You have an active primary care
team working with you.
30 mins
PART 1
120
Mr Albert Torrens is a 45 year old man whose wife telephones you, his general
practitioner, one morning at 2.00 am. She tells you that he has severe pain in his
stomach and is vomiting.
Q.1
How do you respond?
Ask details of pain - how long, character.
?
known precipitating factor eg food, alcohol
?
content of vomitus - eg blood
?
similar past medical history
?
current status re severity of pain
Agree to visit.
6
You arrive at the home and find Mr Torrens looking shocked, pale and sweating.
He is sitting upright, leaning forwards.
Q.2
What further questions would you ask?
Further details of pain - exact site, radiation, character,
duration, precipitating and relieving factors
Vomitus - content, volume
Earlier food & drink consumption
Any relevant contact
Similar episode in past
Recent bowel movement
6
Q.3
What are your initial major differential diagnoses for a
patient with severe non-colicky epigastric pain? List 3.
Acute duodenal ulcer - perforation
Myocardial infarction
Acute pancreatitis
3
119
Q.4
Describe your examination of Mr Torrens.
Observe - colour, respiration, general condition
CVS - BP: Pulse: HS
RS - auscultate lung fields
Abs - observe: palpate: percuss: auscultate
- consider PR
max 10
You admit Mr Torrens to the local hospital as a surgical emergency.
Q.5
What investigations do you think should be done when he
arrives there?
A.
Bloods -
(i)
(ii)
(iii)
(iv)
(v)
FBP, including WCC PCV
U&E
LFT, including calcium, glucose
Serum amylase
If shocked - blood gases
B.
C.
D.
Urinalysis
ECG
XR - Straight XR Abd; ?CXR
max 8
After investigations Mr Torrens is diagnosed as having acute pancreatitis.
Q.6
What are the common causes for this condition? List 4.
1.
2.
3.
4.
Gallstones/biliary tract disease
Alcohol
Post trauma esp. abdominal surgery
Viral
4
Mr Torrens settles with conservative management and is discharged home a week
later. His wife and 15 year old daughter, Alison, bring his discharge note to the
surgery. Mrs Torrens mentions that Alison, who has Down's Syndrome, has
developed a cough and seems to be short of breath with exertion.
Q.7
What salient features would you wish to consider in taking
a history and examining Alison?
Physical - details of general health, other symptoms,
Cough
?productive
duration
relation to exercise
?wheeze associated
Psychological
-
effect of father's illness
possible adolescent problems
120
Social
-
?schooling/prospect of changing
to work situation
10
You diagnose that she has a chest infection and decide to give her an antibiotic.
Q.8
What specific question will you ask before writing the
prescription?
Has she any allergies?
Q.9
1
Write a prescription for an antibiotic for Alison.
eg
Amoxycillin
specify caps / tabs / syrup
Correct name
Specific direction to take
Quantity specified.
8
Mr Torrens brings Alison back for review two weeks later. She is well again but he
complains of vague right sided pain and tells you that his urine has become very
dark.
Q.10
What do you specifically look for on clinical examination?
Icterus
Hepar - tender +/- splenomegaly
(possible sign of chronic liver disease
eg spider naevi, etc)
4
You check his liver function tests which confirm that he has acute hepatitis. He
admits to a recent heavy consumption of alcohol.
Q.11
What further questions would you ask him?
Amount
how much, how often
When, where, why - circumstances of drinking
Ever felt he should cut down?
Ever been annoyed by people criticising his drinking?
Ever felt guilty?
Ever had to drink first thing in morning
Memory blanks
Physical effects, including trauma
121
Legal offences
Problems at work
Problems with finances
Problems with family
Ever secret drinking?
10
He returns to see you 4 weeks later with obvious weight loss and deep jaundice.
You arrange his admission to the local hospital's medical ward.
Q.12
What investigations would you expect to be done and what
abnormalities would support the diagnosis of alcoholic
hepatitis?
FBP increased MCU
LFT increased GT, AST, ALT
Ultrasound abdomen
other diagnosis indicated
Coagulation screen, including PTI : hope to find named?
Liver biopsy, fatty necrosis
10
Q.13
How would you expect him to be treated?
Alcohol withdrawn
Vitamin supplements
Sedation eg chlordiazepoxide if develop DT's
After support/referred for alcohol addiction
4
A week later he is discharged and you visit him at home.
Q.14
What topics would you cover during the consultation?
His awareness of problem
Motivation - attitude to problem, to alcohol, to prospect of
change
General health - appetite, weight
Attitude to and of family
Financial and work situation
Goals - plans for future - involvement of help eg AA,
alcohol addiction teams, etc.
6
122
Over the next 6 months you see Mr Torrens several times, he claims to feel well
and to continue to abstain from alcohol. Then, during morning surgery, you receive
a phone call from Mrs Torrens. She has found her husband lying on the floor and
cannot rouse him.
Q.15
What questions do you ask her on the phone?
Any response from him at all?
Is he breathing OK?
Evidence of injury?
What does she think has happened?
4
You go and see him. He is obviously jaundiced and smells of alcohol. He is
responsive to painful stimuli.
Q.16
How do you manage the situation?
Ask Mrs Torrens about his recent health & behaviour
Examine him physically
Arrange hospital admission
Suggest Mrs Torrens should accompany him
Enquire about Alison's care if she does go to the hospital
Allow Mrs Torrens to express her fears, questions, feelings
Explain possible causes eg fall, introxication, other
reasons need to be excluded ?head injury.
6
In hospital his level of consciousness rises but despite treatment he develops
fulminant liver failure.
Q.17
What complications do you expect to develop?
Mental confusion - deepening coma
Increasing jaundice
Flapping tremor
Bleeding - purpura, haematemesis
4
Ten days later Mr Torrens dies. You are informed of his death by hospital staff
who tell you Mrs Torrens is very distressed. You go and visit her in her home.
Q.18 How would you plan to manage the consultation?
Allow her to ventilate
Be aware of bereavement reaction, especially discuss possible
feelings of guilt, blame, anger and resentment
Fears for future - especially with regard to Alison
Discuss how to tell Alison about father's death
Ensure available practical help for present
Assure of continued support and availability
12
123
PART 2
68
Mr Brown (aged 35) and his son, John (aged 4/12) arrive for their appointment.
The health visitor has diagnosed that John has atopic eczema affecting mostly his
face and flexors.
30 mins
Q.1
What advice would you give Mr Brown on
(a)
Prognosis
No cure. 90% grow out of it.
(b)
2)
)
)
Non pharmacological advice. List 6 areas.
)
)
)
)
Avoid soap/perfumed products
Don't overheat
Synthetic material filled pillows/duvets
Cotton clothing
Pre/post swimming with vaseline
Avoid biological washing powders
Avoid pets
)
)
)
)
) max
14
Minimise house dust
National Eczema Society
(c)
)
6
)
)
Steroids. List 6 areas.
)
)
Only when necessary
Use sparingly
Only on eczematous skin
Not on face (other than 1% HC)
Follow doctor's instructions
Report back for regular review
How much to apply
What to do if condition worsens
)
)
)
)
)
)
6
)
)
124
Mr Brown asks for his blood pressure to be taken. Your nurse had taken it that
morning obtaining a reading of 180/100. You obtain a pressure of 150/95.
Q.2
Give reasons as to why the two readings differ. List 5.
-
Patient more relaxed
Size of cuff
Different syphgmomanometer
Position of arm
Patient lying / standing
Kortacoffs sounds 4th / 5th
5
Q.3
Describe specifically your examination of Mr Brown.
Heart size
HT
Lungs for creps
Heart for murmurs, eg coarctation
Abdomen for kidneys
Fundi to grade degree of
Femorals
6
Q.4
What simple tests would you carry out?
-
Q.5
3
Discuss non-pharmacological management and advice with
Mr Brown.
-
Q.6
Urine for protein and sugar
U & E for renal function
ECG to determine if LVH
Non smoking
Minimise alcohol consumption
Regular exercise
No salt to diet
Diet to achieve optimal BMI
10
If Mr Brown's pressure did not settle suggest a pharmacological
plan of therapy.
-
Diuretic therapy eg Bendrofluo\ide 2.5mg daily
Beta-blocking drugs eg Atenolol 50mg daily
Calcium-channel blockers eg Nifedipine 30mg daily
Ace inhibitors eg Captopril 12.5mg bd
16
125
Your nurse says that she would like an HIV test.
Q.7
What areas would you cover in your pre-test counselling. List 10.
What exactly the test is including seroconversion?
The difference between HIV and AIDS
Medical benefit of knowing HIV status
How HIV infection is transmitted
Support servides in event of a possible infection
Medical confidentiality
Is more time needed to think it over?
Does the patient need to see a counsellor?
How she will get the result
Employment issues
PART 3
10
80
Marion Green, who is twenty, comes to see you almost in tears as she has
frequency and dysuria yet again. Since she became sexually active 18 months
previously the problem has become worse. She has had about eight episodes in
the last year. When the problem started MSU's were regularly done and coliform
infections identified on nearly all occasions. The symptoms do respond to
antibiotics but have recurred so frequently she now dreads intercourse and this is
producing difficulties with her present boy-friend.
Q.1
What areas would you wish to discuss with your patient?
1.
2.
3.
4.
5.
6.
Q.2
Highly unlikely to be serious condition. Reassurance.
Need for laboratory test - MSU
Sexual history - difficulties.
Explanation of how
sexual activity increases the risk of UTI occurrence
and simple advice to counteract this.
Contraception
Treatment
Referral for investigation
6
Apart from E.coli what other organisms are implicated in Urinary
Tract Infection? List 4.
Staphylococcus
Klebsiella
Proteus
Pseudomonas
4
126
Q.3
Q.4
Q.5
Which findings on dipstick analysis are a good indicator of
Urinary Tract Infection.
Protein
Blood
Nitrite
Leucocyte esterase
4
How would you propose to manage the condition on this
occasion? Give the dosage of drugs you might use.
Increased fluid intake
Alkalinizing agent (pot cit / sod bic)
Single dose antibiotic
3 day course
Trimethoprim 200 mg bd x 3 days
Amoxicillin 250 mg tid x 3 days
Cephaloxin 250 mg
8
What other prophylactic medicine might you also prescribe at this
time?
Canesten pessary to prevent vaginal thrush.
2
Marion returns in 10 days and tells you her symptoms have cleared up as is usual,
however, she is worried about recurrence.
Q.6
How would you plan to manage the patient at this stage?
Repeat advice on prevention
Prophylactic antibiotic treatment using either
Trimethoprim or nitrofurantoin
Referral for investigation in case she has a
congenital abnormality or a simple
correctable condition eg urethral stricture
6
Several years pass before you see your patient again. She had left your practice
but has now returned as a young wife and mother. Her husband is a local
business man who travels on business a lot. She asks you for a prescription for
"thrush" and tells you she has had several episodes in the last year and it usually
responds to Canesten.
127
Q.7
Is investigation essential at this stage? Give your reasons for
investigating or not investigating. If you do decide to investigate
what would you do?
Not really necessary. Classical symptoms. No
possibility of STDs.
Should test urine for sugar
As she is "new" patient might carry out speculum
examination and take a swab. Also because it
would provide opportunity to ensure cervical
smear status up to date.
Q.8
12
What organisms infecting the female genital tract can and cannot
be isolated with certainty in the setting of General Practice?
High vaginal swab will pick up Trichomonas, Candida
and Gardnerella and Group B Streptococci. 1 each = 4
Chlamydia and neisseria gonorrhoea difficult to
culture in GP.
1 each = 2
An endocervical swab and urethral swab in special
transport medium with good access to a local
laboratory are essential.
1 each = 4
)
)
)
)
) 10
)
)
)
)
As expected Candida is found in this case.
Q.9
What treatment would you prescribe for this episode and what
strategies might be adopted to ease her recurrent attacks?
Clotrimazole either as single dose vaginal tablet or a
3 night course of 200 mg. Cream may also be used.
4
Oral antifungals eg fluconazole 150 mg may be used
as an alternative.
2
12
-
Ensure diagnosis is correct - HVS
Ensure compliance with treatment
Check for predisposing factors - diabetes, Fe anaemia,
pregnancy, antibiotics, steroid therapy, thyroid disorders
Investigate partner
Prophylactic treatment
6
128
PART 4
66
Mrs Bailey, a 45 year old school teacher brings her son John to see you. He is
eight years old. The presenting complaint is one of a persistent dry cough which is
annoying the whole family.
30 mins
Q.1
What areas would you wish to cover in the preliminary history?
More about the cough: how long, character, any partic. time of day 4
How is he otherwise - any fever or constitutional upset
2
10
2
Any restriction in activities eg sports
Any family history of asthma
Q.2
2
What examination would you carry out?
General appearance eg
pallor, obesity, chest shape, fever, cyanosis,
any obvious breathing difficulties at rest
Ausculate chest looking for equal air entry and
any evidence of expiratory wheeze
Take a peak flow reading in the surgery
Give child peak flow meter home to do daily
peak flows and review after a week
4
2
2
10
2
You decide that John probably has asthma and give him a salbutamol inhaler to
try.
Q.3
What advice do you give to John and Mrs Bailey and what general aims
should you have in mind when treating children with asthma?
Explanation of the diagnosis and how treatment will help
Need for a regular review particularly in the early stages
to guage appropriate level of therapy
Involve the child (and the family) in the management
Emphasise the importance of adequate treatment (partic.
parents should not be afraid of medications)
Aim to abolish symptoms
Importance of allowing normal growth in children
Aim for child to be able to participate in normal activities
(eg sport, PE) and to minimise absence from school
Avoidance of identified causes where possible
2
2
2
2
2
1
1
1
14
129
Aim to use the lowest effective doses of convenient
medications minimising short and long term
side effects
1
You ask John to come back for a review in one week's time, however, he does not
attend for another month or so. The salbutamol inhaler worked very well initially,
however, over the past week he has been getting more and more wheezy and out
of breath. Two days ago it was so bad he ended up in the A&E department of the
local hospital and was given nebulised salbutamol which rapidly alleviated his
symptoms.
Q.4
What might you include in your management now?
Reiterate importance of regular review
Prescribe regular inhaled anti-inflammatory agents such as
cromoglycate and be prepared to step up to inhaled
steroids (beclomethasone or budesonide) sooner
rather than later.
Short acting beta-antagonists should ony be used
as required for symptom relief.
Address any concerns or fears in John or his parents
Arrange review
2
12
4
2
2
2
John does well on his regular inhaled steroid. However, several months later you
are called out in the middle of the night to his house. He had just returned from a
school camping trip that evening when his asthma became much worse. Mrs
Bailey is very anxious and says he is having his worst ever attack. You decide to
call round immediately.
Q.5
Q.6
What are the features of a severe asthmatic attack in a child?
Too breathless to talk
Too breathless to feed
Respiratory rate > 50 per min
Heart rate > 140 per min
PEF < 50% of best previous result
Cyanosis
Silent chest or feeble respiratory effort
Fatigue or exhaustion
Agitation or reduced level of consciousness
1
1
2
2
2 16
2
2
2
2
What treatment would you consider for a severe asthmatic episode?
Oxygen via face mask
Salbutamol or terbutaline via a nebuliser
Prednisolone tabs 1 - 2 mg/kg body weight up to 40 mg
IV hydrocortisone
IV aminophylline
4